Exam2Medsurg

Ace your homework & exams now with Quizwiz!

Enlarged liver size Ascites Hemorrhoids

A client is given a diagnosis of hepatic cirrhosis. The client asks the nurse what findings led to this determination. Which of the following clinical manifestations would the nurse correctly identify? Select all that apply.

Metoclopramide

A client with GERD has undergone diagnostic testing and it has been determined that increasing the pace of gastric emptying may help alleviate symptoms. The nurse should anticipate that the client may be prescribed what drug?

Administering diuretics Implementing fluid restrictions Enhancing client positioning

A client with end-stage liver disease has developed hypervolemia. What nursing interventions would be most appropriate when addressing the client's fluid volume excess? Select all that apply.

acute cholecystitis

A middle-aged obese female presents to the ED with severe radiating right-sided flank pain, nausea, vomiting, and fever. A likely cause of these symptoms is:

Ask the client to remain inactive for 5 minutes.

A nurse applies an ostomy appliance to a client who is recovering from ileostomy surgery. Which intervention should the nurse utilize to prevent leakage from the appliance?

Early diagnosis and treatment of gastroesophageal reflux disease

A nurse is addressing the prevention of esophageal cancer in response to a question posed by a participant in a health promotion workshop. What action should the nurse recommend as having the greatest potential to prevent esophageal cancer?

Diarrhea Tachycardia Diaphoresis

A nurse is assessing a client receiving tube feedings and suspects dumping syndrome. What would lead the nurse to suspect this? Select all that apply

Encourage the family to bring in the client's favorite foods.

A nurse is caring for a client who is postoperative from a neck dissection. What would be the mostappropriate nursing action to enhance the client's appetite?

A) Keep the patients hips in abduction at all times.

A nurse is caring for a patient who is postoperative day 1 right hip replacement. How should the nurse position the patient? A) Keep the patients hips in abduction at all times. B) Keep hips flexed at no less than 90 degrees. C) Elevate the head of the bed to high Fowlers. D) Seat the patient in a low chair as soon as possible.

B) Risk for Infection Related to the Presence of a Subclavian Catheter

A nurse is caring for a patient with a subclavian central line who is receiving parenteral nutrition (PN). In preparing a care plan for this patient, what nursing diagnosis should the nurse prioritize? A) Risk for Activity Intolerance Related to the Presence of a Subclavian Catheter B) Risk for Infection Related to the Presence of a Subclavian Catheter C) Risk for Functional Urinary Incontinence Related to the Presence of a Subclavian Catheter D) Risk for Sleep Deprivation Related to the presence of a Subclavian Catheter

B) Report this finding to the primary care provider due to the possibility of hepatic encephalopathy.

A nurse is caring for a patient with cirrhosis secondary to heavy alcohol use. The nurses most recent assessment reveals subtle changes in the patients cognition and behavior. What is the nurses most appropriate response? A) Ensure that the patients sodium intake does not exceed recommended levels. B) Report this finding to the primary care provider due to the possibility of hepatic encephalopathy. C) Inform the primary care provider that the patient should be assessed for alcoholic hepatitis. D) Implement interventions aimed at ensuring a calm and therapeutic care environment.

A) Improving the patients level of function

A nurse is planning the care of a patient who has undergone orthopedic surgery. What main goal should guide the nurses choice of interventions? A) Improving the patients level of function B) Helping the patient come to terms with limitations C) Administering medications safely D) Improving the patients adherence to treatment

C) The patient maintains or gains weight.

A nurse is preparing to discharge a patient after recovery from gastric surgery. What is an appropriate discharge outcome for this patient? A) The patients bowel movements maintain a loose consistency. B) The patient is able to tolerate three large meals a day. C) The patient maintains or gains weight. D) The patient consumes a diet high in calcium.

Atelectasis Pneumonia Hemorrhage

A nurse is providing care for a client who is postoperative day 2 following gastric surgery. The nurse's assessment should be planned in light of the possibility of what potential complications? Select all that apply.

A) Does your pain resolve when you have something to eat?

A patient comes to the clinic complaining of pain in the epigastric region. What assessment question during the health interview would most help the nurse determine if the patient has a peptic ulcer? A) Does your pain resolve when you have something to eat? B) Do over-the-counter pain medications help your pain? C) Does your pain get worse if you get up and do some exercise? D) Do you find that your pain is worse when you need to have a bowel movement?

Clay-colored feces Pruritus Jaundice

A patient is admitted to the hospital with a possible common bile duct obstruction. What clinical manifestations does the nurse understand are indicators of this problem? (Select all that apply.)

D) Maintain the patient in a semi-Fowlers position whenever possible.

A patient is being treated on the acute medical unit for acute pancreatitis. The nurse has identified a diagnosis of Ineffective Breathing Pattern Related to Pain. What intervention should the nurse perform in order to best address this diagnosis? A) Position the patient supine to facilitate diaphragm movement. B) Administer corticosteroids by nebulizer as ordered. C) Perform oral suctioning as needed to remove secretions. D) Maintain the patient in a semi-Fowlers position whenever possible.

C) Compartment syndrome

A patient is complaining of pain in her casted leg. The nurse has administered analgesics and elevated the limb. Thirty minutes after administering the analgesics, the patient states the pain is unrelieved. The nurse should identify the warning signs of what complication? A) Subcutaneous emphysema B) Skin breakdown C) Compartment syndrome D) Disuse syndrome

Pain Erythema Fever

A patient is diagnosed with osteomyelitis of the right leg. What signs and symptoms does the nurse recognize that are associated with this diagnosis? (Select all that apply.)

An abnormal glucose tolerance Glucosuria Hyperglycemia

A patient is suspected to have pancreatic carcinoma and is having diagnostic testing to determine insulin deficiency. What would the nurse determine is an indicator for insulin deficiency in this patient? (Select all that apply).

C) Intermittent pain and bloody stool

A patient is undergoing diagnostic testing for a tumor of the small intestine. What are the most likely symptoms that prompted the patient to first seek care? A) Hematemesis and persistent sensation of fullness B) Abdominal bloating and recurrent constipation C) Intermittent pain and bloody stool D) Unexplained bowel incontinence and fatty stools

Parotid

A patient reports an inflamed salivary gland below the right ear. The nurse documents probable inflammation of which gland?

B) Decubitus ulcer

A patient returns to the floor after a laparoscopic cholecystectomy. The nurse should assess the patient for signs and symptoms of what serious potential complication of this surgery? A) Diabetic coma B) Decubitus ulcer C) Wound evisceration D) Bile duct injury

B) 60 mL of milky or cloudy drainage

A patient returns to the unit after a neck dissection. The surgeon placed a Jackson Pratt drain in the wound. When assessing the wound drainage over the first 24 postoperative hours the nurse would notify the physician immediately for what? A) Presence of small blood clots in the drainage B) 60 mL of milky or cloudy drainage C) Spots of drainage on the dressings surrounding the drain D) 120 mL of serosanguinous drainage

D) Instead of eating three meals a day, try eating smaller amounts more often.

A patient seeking care because of recurrent heartburn and regurgitation is subsequently diagnosed with a hiatal hernia. Which of the following should the nurse include in health education? A) Drinking beverages after your meal, rather than with your meal, may bring some relief. B) Its best to avoid dry foods, such as rice and chicken, because theyre harder to swallow. C) Many patients obtain relief by taking over-the-counter antacids 30 minutes before eating. D) Instead of eating three meals a day, try eating smaller amounts more often.

A) The patients BUN and creatinine levels are within reference range following the CT.

A patient will be undergoing abdominal computed tomography (CT) with contrast. The nurse has administered IV sodium bicarbonate and oral acetylcysteine (Mucomyst) before the study as ordered. What would indicate that these medications have had the desired therapeutic effect? A) The patients BUN and creatinine levels are within reference range following the CT. B) The CT yields high-quality images. C) The patients electrolytes are stable in the 48 hours following the CT. D) The patients intake and output are in balance on the day after the CT.

D) Digestion of proteins

A patient with cystic fibrosis takes pancreatic enzyme replacements on a regular basis. The patients intake of trypsin facilitates what aspect of GI function? A) Vitamin D synthesis B) Digestion of fats C) Maintenance of peristalsis D) Digestion of proteins

AC

A patients assessment and diagnostic testing are suggestive of acute pancreatitis. When the nurse is performing the health interview, what assessment questions address likely etiologic factors? Select all that apply. A) How many alcoholic drinks do you typically consume in a week? B) Have you ever been tested for diabetes? C) Have you ever been diagnosed with gallstones? D) Would you say that you eat a particularly high-fat diet? E) Does anyone in your family have cystic fibrosis?

Exhibiting hemoglobin A1C 8.2

An elderly client states, "I don't understand why I have so many caries in my teeth." What assessment made by the nurse places the client at risk for dental caries?

A) Peritonitis

Diagnostic imaging and physical assessment have revealed that a patient with peptic ulcer disease has suffered a perforated ulcer. The nurse recognizes that emergency interventions must be performed as soon as possible in order to prevent the development of what complication? A) Peritonitis B) Gastritis C) Gastroesophageal reflux D) Acute pancreatitis

ascites

sodium restriction is also indicated to prevent what

esophageal varices

this condition is almost always caused by portal hypertension which results from obstruction of the portal venous circulation within the damaged liver

A) Osteoblasts

A child is growing at a rate appropriate for his age. What cells are responsible for the secretion of bone matrix that eventually results in bone growth? A) Osteoblasts B) Osteocytes C) Osteoclasts D) Lamellae

Cholelithiasis

A client comes to the clinic and informs the nurse that he is there to see the physician for right upper abdominal discomfort, nausea, and frequent belching especially after eating a meal high in fat. What disorder do these symptoms correlate with?

Insertion of an NG tube for decompression

A client has been admitted to the hospital after diagnostic imaging revealed the presence of a gastric outlet obstruction (GOO). What is the nurse's priority intervention?

Encourage the client to connect with a community-based support group.

A client has been treated in the hospital for an episode of acute pancreatitis. The client has acknowledged the role that his alcohol use played in the development of his health problem, but has not expressed specific plans for lifestyle changes. What is the nurse's most appropriate response?

Deep vein thrombosis Compartment syndrome Fat embolism

A client is admitted to the orthopedic unit with a fractured femur after a motorcycle accident. The client has been placed in traction until his femur can be rodded in surgery. For what early complications should the nurse monitor this client? Select all that apply.

Potassium-rich foods

A client is being treated for prolonged diarrhea. Which foods should the nurse encourage the client to consume?

Make appropriate referrals to services that provide psychosocial support.

A client is recovering in the hospital following gastrectomy. The nurse notes that the client has become increasingly difficult to engage and has had several angry outbursts at staff members in recent days. The nurse's attempts at therapeutic dialogue have been rebuffed. What is the nurse's most appropriate action?

"Hydrochloric acid is secreted by glands in the stomach in response to the actual or anticipated presence of food."

A client presents at the ambulatory clinic reporting recurrent sharp stomach pain that is relieved by eating. The nurse suspects that the client may have an ulcer. How should the nurse explain the formation and role of acid in the stomach to the client?

Acute gastritis

A client reports to the clinic, stating that she rapidly developed headache, abdominal pain, nausea, hiccuping, and fatigue about 2 hours ago. For dinner, she ate buffalo chicken wings and beer. Which of the following medical conditions is most consistent with the client's presenting problems?

"I cannot seem to catch my breath." "I have a pins-and-needles sensation in my toes." Dorsiplantar weak and unequal bilaterally T 101.2 degrees F; HR 110; RR 28; pulse oximetry 90%

A client with a fractured femur is admitted to the nursing unit. Which assessment finding requires follow up by the nurse? Select all that apply.

Diarrhea

A client with a gastrojejunostomy is beginning to take solid food. Which finding would lead the nurse to suspect that the client is experiencing dumping syndrome?

Ensure that a large tourniquet is in the room.

A client with a right below-the-knee amputation is being transferred from the postanesthesia care unit to a medical-surgical unit. What is the highest priority nursing intervention by the receiving nurse?

Prepare to remove the cast. Provide support to the injured extremity.

A client with a short arm cast is suspected to have compartment syndrome. What actions should the nurse include in the plan of care? Select all that apply.

diuretics albumin

A client with acute pancreatitis has jaundice with diminished bowel sounds and a tender distended abdomen. Additionally, lab results indicate hypovolemia. What will the physician order to treat the large amount of protein-rich fluid that has been released into the client's tissues and peritoneal cavity? Select all that apply.

phytonadione (Mephyton).

A client with advanced cirrhosis has a prothrombin time (PT) of 15 seconds, compared with a control time of 11 seconds. The nurse expects to administer:

D) Monthly administration of injections of vitamin B12

A community health nurse is preparing for an initial home visit to a patient discharged following a total gastrectomy for treatment of gastric cancer. What would the nurse anticipate that the plan of care is most likely to include? A) Enteral feeding via gastrostomy tube (G tube) B) Gastrointestinal decompression by nasogastric tube C) Periodic assessment for esophageal distension D) Monthly administration of injections of vitamin B12

Hypothyroidism

A group of students are reviewing information about osteoporosis in preparation for a class discussion. The students demonstrate a need for additional review when they state which of the following as a risk factor?

Excrete bile

A group of students is reviewing information about the liver and associated disorders. The group demonstrates understanding of the information when they identify which of the following as a primary function of the liver?

D) Malabsorption and hyperglycemia

A home health nurse is caring for a patient discharged home after pancreatic surgery. The nurse documents the nursing diagnosis Risk for Imbalanced Nutrition: Less than Body Requirements on the care plan based on the potential complications that may occur after surgery. What are the most likely complications for the patient who has had pancreatic surgery? A) Proteinuria and hyperkalemia B) Hemorrhage and hypercalcemia C) Weight loss and hypoglycemia D) Malabsorption and hyperglycemia

Stage 2

A nurse assesses a patient diagnosed with hepatic encephalopathy. She observes a number of clinical signs, including asterixis and fetor hepaticus; the patient's electroencephalogram (EEG) is abnormal. The nurse documents that the patient is exhibiting signs of which stage of hepatic encephalopathy?

Fear of eating

A nurse cares for a client who is postoperative bariatric surgery and has experienced frequent episodes of dumping syndrome. The client now reports anorexia. What is the primary reason for the client's report of anorexia?

D) Report this finding to the patients primary care provider.

A nurse has obtained an order to remove a patients NG tube and has prepared the patient accordingly. After flushing the tube and removing the nasal tape, the nurse attempts removal but is met with resistance. Because the nurse is unable to overcome this resistance, what is the most appropriate action? A) Gently twist the tube before pulling. B) Instill a digestive enzyme solution and reattempt removal in 10 to 15 minutes. C) Flush the tube with hot tap water and reattempt removal. D) Report this finding to the patients primary care provider.

B) Medulla oblongata

A nurse in a stroke rehabilitation facility recognizes that the brain regulates swallowing. Damage to what area of the brain will most affect the patients ability to swallow? A) Temporal lobe B) Medulla oblongata C) Cerebellum D) Pons

B) Use warm saline to rinse the mouth as needed.

A nurse in an oral surgery practice is working with a patient scheduled for removal of an abscessed tooth. When providing discharge education, the nurse should recommend which of the following actions? A) Rinse the mouth with alcohol before bedtime for the next 7 days. B) Use warm saline to rinse the mouth as needed. C) Brush around the area with a firm toothbrush to prevent infection. D) Use a toothpick to dislodge any debris that gets lodged in the socket.

C) Early diagnosis and treatment of gastroesophageal reflux disease

A nurse is addressing the prevention of esophageal cancer in response to a question posed by a participant in a health promotion workshop. What action has the greatest potential to prevent esophageal cancer? A) Promotion of a nutrient-dense, low-fat diet B) Annual screening endoscopy for patients over 50 with a family history of esophageal cancer C) Early diagnosis and treatment of gastroesophageal reflux disease D) Adequate fluid intake and avoidance of spicy foods

A) Premature removal of the G tube

A nurse is admitting a patient to the postsurgical unit following a gastrostomy. When planning assessments, the nurse should be aware of what potential postoperative complication of a gastrostomy? A) Premature removal of the G tube B) Bowel perforation C) Constipation D) Development of peptic ulcer disease (PUD)

A) Fried chicken

A nurse is assisting with serving dinner trays on the unit. Upon receiving the dinner tray for a patient admitted with acute gallbladder inflammation, the nurse will question which of the following foods on the tray? A) Fried chicken B) Mashed potatoes C) Dinner roll D) Tapioca pudding

Strategies for avoiding irritating foods and beverages

A nurse is caring for a client hospitalized with an exacerbation of chronic gastritis. What health promotion topic should the nurse emphasize?

The use of moderate sedation

A nurse is caring for a client who has been scheduled for endoscopic retrograde cholangiopancreatography (ERCP) the following day. When providing anticipatory guidance for this client, the nurse should describe what aspect of this diagnostic procedure?

D) Strategies for avoiding irritating foods and beverages

A nurse is caring for a patient hospitalized with an exacerbation of chronic gastritis. What health promotion topic should the nurse emphasize? A) Strategies for maintaining an alkaline gastric environment B) Safe technique for self-suctioning C) Techniques for positioning correctly to promote gastric healing D) Strategies for avoiding irritating foods and beverages

B) Promptly report these indications of venous congestion.

A nurse is caring for a patient who has undergone neck resection with a radial forearm free flap. The nurses most recent assessment of the graft reveals that it has a bluish color and that mottling is visible. What is the nurses most appropriate action? A) Document the findings as being consistent with a viable graft. B) Promptly report these indications of venous congestion. C) Closely monitor the patient and reassess in 30 minutes. D) Reposition the patient to promote peripheral circulation.

D) Inadequate nutrition and decreased saliva production can cause cavities

A nurse is caring for a patient who is acutely ill and has included vigilant oral care in the patients plan of care. Why are patients who are ill at increased risk for developing dental caries? A) Hormonal changes brought on by the stress response cause an acidic oral environment B) Systemic infections frequently migrate to the teeth C) Hydration that is received intravenously lacks fluoride D) Inadequate nutrition and decreased saliva production can cause cavities

B) Erosion of the lining of the stomach or intestine

A nurse is caring for a patient who just has been diagnosed with a peptic ulcer. When teaching the patient about his new diagnosis, how should the nurse best describe a peptic ulcer? A) Inflammation of the lining of the stomach B) Erosion of the lining of the stomach or intestine C) Bleeding from the mucosa in the stomach D) Viral invasion of the stomach wall

C) Smokes one pack of cigarettes daily.

A nurse is completing a health history on a patient whose diagnosis is chronic gastritis. Which of the data should the nurse consider most significantly related to the etiology of the patients health problem? A) Consumes one or more protein drinks daily. B) Takes over-the-counter antacids frequently throughout the day. C) Smokes one pack of cigarettes daily. D) Reports a history of social drinking on a weekly basis.

C) Bed rest lowers the metabolic rate and reduces enzyme production.

A nurse is creating a care plan for a patient with acute pancreatitis. The care plan includes reduced activity. What rationale for this intervention should be cited in the care plan? A) Bed rest reduces the patients metabolism and reduces the risk of metabolic acidosis. B) Reduced activity protects the physical integrity of pancreatic cells. C) Bed rest lowers the metabolic rate and reduces enzyme production. D) Inactivity reduces caloric need and gastrointestinal motility.

C) Limiting intake of alcohol

A nurse is educating a patient with gout about lifestyle modifications that can help control the signs and symptoms of the disease. What recommendation should the nurse make? A) Ensuring adequate rest B) Limiting exposure to sunlight C) Limiting intake of alcohol D) Smoking cessation

D) Inform the surgeon of this finding.

A nurse is emptying an orthopedic surgery patients closed suction drainage at the end of a shift. The nurse notes that the volume is within expected parameters but that the drainage has a foul odor. What is the nurses best action? A) Aspirate a small amount of drainage for culturing. B) Advance the drain 1 to 1.5 cm. C) Irrigate the drain with normal saline. D) Inform the surgeon of this finding.

A,B,C

A nurse is explaining a patients decreasing bone density in terms of the balance between bone resorption and formation. What dietary nutrients and hormones play a role in the resorption and formation of adult bones? Select all that apply. A) Thyroid hormone B) Growth hormone C) Estrogen D) Vitamin B12 E) Luteinizing hormone

Tachycardia Hypotension A rigid, board-like abdomen

A nurse is monitoring a client with peptic ulcer disease. Which assessment findings would most likely indicate perforation of the ulcer? Select all that apply.

D) A slightly decreased size of the liver

A nurse is performing an admission assessment for an 81-year-old patient who generally enjoys good health. When considering normal, age-related changes to hepatic function, the nurse should anticipate what finding? A) Similar liver size and texture as in younger adults B) A nonpalpable liver C) A slightly enlarged liver with palpably hard edges D) A slightly decreased size of the liver

D) Imbalanced Nutrition: Less Than Body Requirements

A nurse is performing health education with a patient who has a history of frequent, serious dental caries. When planning educational interventions, the nurse should identify a risk for what nursing diagnosis? A) Ineffective Tissue Perfusion B) Impaired Skin Integrity C) Aspiration D) Imbalanced Nutrition: Less Than Body Requirements

A) Watery with blood and mucus

A nurse is preparing to provide care for a patient whose exacerbation of ulcerative colitis has required hospital admission. During an exacerbation of this health problem, the nurse would anticipate that the patients stools will have what characteristics? A) Watery with blood and mucus B) Hard and black or tarry C) Dry and streaked with blood D) Loose with visible fatty streaks

A) High levels of alcohol consumption

A nurse is presenting an educational event to a local community group. When speaking about colorectal cancer, what risk factor should the nurse cite? A) High levels of alcohol consumption B) History of bowel obstruction C) History of diverticulitis D) Longstanding psychosocial stress

Vasomotor symptoms associated with dumping syndrome

A nurse is providing care for a client recovering from gastric bypass surgery. During assessment, the client exhibits pallor, perspiration, palpitations, headache, and feelings of warmth, dizziness, and drowsiness. The client reports eating 90 minutes ago. What will the nurse suspect?

A) Patient will accurately identify foods that trigger symptoms.

A nurse is providing care for a patient who has a diagnosis of irritable bowel syndrome (IBS). When planning this patients care, the nurse should collaborate with the patient and prioritize what goal? A) Patient will accurately identify foods that trigger symptoms. B) Patient will demonstrate appropriate care of his ileostomy. C) Patient will demonstrate appropriate use of standard infection control precautions. D) Patient will adhere to recommended guidelines for mobility and activity.

BCD

A nurse is providing care for a patient who is postoperative day 2 following gastric surgery. The nurses assessment should be planned in light of the possibility of what potential complications? Select all that apply. A) Malignant hyperthermia B) Atelectasis C) Pneumonia D) Metabolic imbalances E) Chronic gastritis

dumping syndrome

A nurse is providing follow-up teaching at a clinic visit for a client recovering from gastric resection. The client reports sweating, diarrhea, nausea, palpitations, and the desire to lie down 15 to 30 minutes after meals. Based on the client's assessment, what will the nurse suspect?

D) Maintain an open dialogue with the patient and facilitate a referral to the wound-ostomy-continence (WOC) nurse.

A nurse is talking with a patient who is scheduled to have a hemicolectomy with the creation of a colostomy. The patient admits to being anxious, and has many questions concerning the surgery, the care of a stoma, and necessary lifestyle changes. Which of the following nursing actions is most appropriate? A) Reassure the patient that the procedure is relatively low risk and that patients are usually successful in adjusting to an ostomy. B) Provide the patient with educational materials that match the patients learning style. C) Encourage the patient to write down these concerns and questions to bring forward to the surgeon. D) Maintain an open dialogue with the patient and facilitate a referral to the wound-ostomy-continence (WOC) nurse.

A) I have this ringing in my ears that just wont go away.

A nurse is working with a patient with rheumatic disease who is being treated with salicylate therapy. What statement would indicate that the patient is experiencing adverse effects of this drug? A) I have this ringing in my ears that just wont go away. B) I feel so foggy in the mornings and it takes me so long to wake up. C) When I eat a meal thats high in fat, I get really nauseous. D) I seem to have lost my appetite, which is unusual for me.

Remove gas and fluids from the stomach Diagnose gastrointestinal motility disorders Flush ingested toxins from the stomach Administer nutritional substances

A nursing instructor is preparing a class about gastrointestinal intubation. Which of the following would the instructor include as reason for this procedure? Select all that apply.

Ingestion of strong acids Irritating foods Overuse of aspirin

A nursing student is caring for a client with gastritis. Which of the following would the student recognize as a common cause of gastritis? Choose all that apply.

A) Glucose tolerance test

A patient has a recent diagnosis of chronic pancreatitis and is undergoing diagnostic testing to determine pancreatic islet cell function. The nurse should anticipate what diagnostic test? A) Glucose tolerance test B) ERCP C) Pancreatic biopsy D) Abdominal ultrasonography

Surgical removal of the diverticulum

A patient has been diagnosed with Zenker's diverticulum. What treatment does the nurse anticipate educating the patient about?

A) Have the patient refrain from food and fluids after midnight.

A patient has been scheduled for an ultrasound of the gallbladder the following morning. What should the nurse do in preparation for this diagnostic study? A) Have the patient refrain from food and fluids after midnight. B) Administer the contrast agent orally 10 to 12 hours before the study. C) Administer the radioactive agent intravenously the evening before the study. D) Encourage the intake of 64 ounces of water 8 hours before the study.

D) Encourage the patient to connect with a community-based support group.

A patient has been treated in the hospital for an episode of acute pancreatitis. The patient has acknowledged the role that his alcohol use played in the development of his health problem, but has not expressed specific plans for lifestyle changes after discharge. What is the nurses most appropriate response? A) Educate the patient about the link between alcohol use and pancreatitis. B) Ensure that the patient knows the importance of attending follow-up appointments. C) Refer the patient to social work or spiritual care. D) Encourage the patient to connect with a community-based support group.

C) Cortical bone

A patient injured in a motor vehicle accident has sustained a fracture to the diaphysis of the right femur. Of what is the diaphysis of the femur mainly constructed? A) Epiphyses B) Cartilage C) Cortical bone D) Cancellous bone

Ultrasonography

A patient is admitted to the hospital with possible cholelithiasis. What diagnostic test of choice will the nurse prepare the patient for?

B) Shuffling gait

A patient is receiving ongoing nursing care for the treatment of Parkinsons disease. When assessing this patients gait, what finding is most closely associated with this health problem? A) Spastic hemiparesis gait B) Shuffling gait C) Rapid gait D) Steppage gait

Hang a solution of dextrose 10% and water until the new solution is available.

A patient is receiving parenteral nutrition. The current solution is nearing completion, and a new solution is to be hung, but it has not arrived from the pharmacy. Which action by the nurse would be most appropriate?

D) Arthocentesis

A patient is suspected of having rheumatoid arthritis and her diagnostic regimen includes aspiration of synovial fluid from the knee for a definitive diagnosis. The nurse knows that which of the following procedures will be involved? A) Angiography B) Myelography C) Paracentesis D) Arthocentesis

D) Gangrene of the gallbladder

A patient presents to the emergency department (ED) complaining of severe right upper quadrant pain. The patient states that his family doctor told him he had gallstones. The ED nurse should recognize what possible complication of gallstones? A) Acute pancreatitis B) Atrophy of the gallbladder C) Gallbladder cancer D) Gangrene of the gallbladder

A) Infection with Helicobacter pylori

A patient presents to the walk-in clinic complaining of vomiting and burning in her mid-epigastria. The nurse knows that in the process of confirming peptic ulcer disease, the physician is likely to order a diagnostic test to detect the presence of what? A) Infection with Helicobacter pylori B) Excessive stomach acid secretion C) An incompetent pyloric sphincter D) A metabolic acidbase imbalance

Covering the area with a clean dressing if the fracture is open Immobilizing the affected site Splinting the injured limb

A patient sustains an open fracture of the left arm after an accident at the roller skating rink. What does emergency management of this fracture involve? (Select all that apply.)

Achalasia

A patient tells the nurse that it feels like food is "sticking" in the lower portion of the esophagus. What motility disorder does the nurse suspect these symptoms indicate?

C) Rigidity of the abdomen

A patient who had surgery for gallbladder disease has just returned to the postsurgical unit from postanesthetic recovery. The nurse caring for this patient knows to immediately report what assessment finding to the physician? A) Decreased breath sounds B) Drainage of bile-colored fluid onto the abdominal dressing C) Rigidity of the abdomen D) Acute pain with movement

C) Promoting maximum shoulder function

A patient who has had a radical neck dissection is being prepared for discharge. The discharge plan includes referral to an outpatient rehabilitation center for physical therapy. What would the goals of physical therapy for this patient include? A) Muscle training to relieve dysphagia B) Relieving nerve paralysis in the cervical plexus C) Promoting maximum shoulder function D) Alleviating achalasia by decreasing esophageal peristalsis

ACE

A patient with end-stage liver disease has developed hypervolemia. What nursing interventions would be most appropriate when addressing the patients fluid volume excess? Select all that apply. A) Administering diuretics B) Administering calcium channel blockers C) Implementing fluid restrictions D) Implementing a 1500 kcal/day restriction E) Enhancing patient positioning

B) I will take this medication for 2 weeks and then gradually stop taking it.

A patient with gallstones has been prescribed ursodeoxycholic acid (UDCA). The nurse understands that additional teaching is needed regarding this medication when the patient states: A) It is important that I see my physician for scheduled follow-up appointments while taking this medication. B) I will take this medication for 2 weeks and then gradually stop taking it. C) If I lose weight, the dose of the medication may need to be changed. D) This medication will help dissolve small gallstones made of cholesterol.

A) Does anyone in your family have experience at giving injections?

A patient with gastritis required hospital treatment for an exacerbation of symptoms and receives a subsequent diagnosis of pernicious anemia due to malabsorption. When planning the patients continuing care in the home setting, what assessment question is most relevant? A) Does anyone in your family have experience at giving injections? B) Are you going to be anywhere with strong sunlight in the next few months? C) Are you aware of your blood type? D) Do any of your family members have training in first aid? Ans: A

D) Preparing the patient for surgical bowel resection

A patients large bowel obstruction has failed to resolve spontaneously and the patients worsening condition has warranted admission to the medical unit. Which of the following aspects of nursing care is most appropriate for this patient? A) Administering bowel stimulants as ordered B) Administering bulk-forming laxatives as ordered C) Performing deep palpation as ordered to promote peristalsis D) Preparing the patient for surgical bowel resection

A) The patients swallowing ability

A patients neck dissection surgery resulted in damage to the patients superior laryngeal nerve. What area of assessment should the nurse consequently prioritize? A) The patients swallowing ability B) The patients ability to speak C) The patients management of secretions D) The patients airway patency

ACD

A patients physician has ordered a liver panel in response to the patients development of jaundice. When reviewing the results of this laboratory testing, the nurse should expect to review what blood tests? Select all that apply. A) Alanine aminotransferase (ALT) B) C-reactive protein (CRP) C) Gamma-glutamyl transferase (GGT) D) Aspartate aminotransferase (AST) E) B-type natriuretic peptide (BNP)

Increased fiber intake Reduced fat intake

An adult client has been diagnosed with diverticular disease after ongoing challenges with constipation. The client will be treated on an outpatient basis. What components of treatment should the nurse anticipate? Select all that apply.

Fluids must be increased to facilitate the evacuation of the stool.

An adult client is scheduled for an upper GI series that will use a barium swallow. What teaching should the nurse include when the client has completed the test?

BD

An adult patient has been diagnosed with diverticular disease after ongoing challenges with constipation. The patient will be treated on an outpatient basis. What components of treatment should the nurse anticipate? Select all that apply. A) Anticholinergic medications B) Increased fiber intake C) Enemas on alternating days D) Reduced fat intake E) Fluid reduction

D) Kyphosis

An older adult patient has come to the clinic for a regular check-up. The nurses initial inspection reveals an increased thoracic curvature of the patients spine. The nurse should document the presence of which of the following? A) Scoliosis B) Epiphyses C) Lordosis D) Kyphosis

The distance measured from the tip of the nose (N) to the earlobe (E) and from the earlobe to the xiphoid (X) process

Before inserting a gastric or enteric tube, the nurse determines the length of tubing that will be needed to reach the stomach or small intestine. The Levin tube, a commonly used nasogastric tube, has circular markings at specific points. This tube should be inserted to 6 to 10 cm beyond what length?

A) For many people, lack of nutrition can cause a loss of bone density.

Diagnostic tests show that a patients bone density has decreased over the past several years. The patient asks the nurse what factors contribute to bone density decreasing. What would be the nurses best response? A) For many people, lack of nutrition can cause a loss of bone density. B) Progressive loss of bone density is mostly related to your genes. C) Stress is known to have many unhealthy effects, including reduced bone density. D) Bone density decreases with age, but scientists are not exactly sure why this is the case.

Bowel perforation

During a colonoscopy with moderate sedation, the patient groans with obvious discomfort and begins bleeding from the rectum. The patient is diaphoretic and has an increase in abdominal girth from distention. What complication of this procedure is the nurse aware may be occurring?

Petechiae possibly due to a transient thrombocytopenia Substernal chest pain Hypoxia

Fat emboli are a major cause of death for patients with fractures. What are the significant signs and symptoms? Select all that apply.

There is a deficiency of activated vitamin D (calcitriol). Calcium and phosphate are not moved to the bones. The bone mass is structurally weaker, and bone deformities occur.

Identify descriptors of the pathophysiologic process seen in osteomalacia. Select all that apply.

Aldactone, an aldosterone-blocking agent would be used. Daily salt intake would be restricted to 2 grams or less. The diuretic will be held if the serum sodium level decreases to <134 m Eq/L.

Management of a patient with ascites includes nutritional modifications and diuretic therapy. Which of the following interventions would a nurse expect to be part of patient care? Select all that apply.

ASR Aspartate aminotransferase

Not specific to the liver but sig elevated with metastatic disease to the liver

C) Determining the patients ability to understand and cooperate with the procedure

Prior to a patients scheduled jejunostomy, the nurse is performing the preoperative assessment. What goal should the nurse prioritize during the preoperative assessment? A) Determining the patients nutritional needs B) Determining that the patient fully understands the postoperative care required C) Determining the patients ability to understand and cooperate with the procedure D) Determining the patients ability to cope with an altered body image

chronic pancreatitis

Progressive inflammatory disorder with struction of the pancreas, cells are replaced by fibrous tissue, pressure within the pancreas increases, obstructing the pancreatic and common bile ducts

B) Keeping the head of the bed slightly elevated

Results of a patient barium swallow suggest that the patient has GERD. The nurse is planning health education to address the patients knowledge of this new diagnosis. Which of the following should the nurse encourage? A) Eating several small meals daily rather than 3 larger meals B) Keeping the head of the bed slightly elevated C) Drinking carbonated mineral water rather than soft drinks D) Avoiding food or fluid intake after 6:00 p.m.

1 hour

Semi-Fowler position is maintained for at least which timeframe following completion of an intermittent tube feeding?

Clay-colored stools Dark urine Jaundice Pruritis

Sixty to eighty percent of pancreatic tumors occur in the head of the pancreas. Tumors in this region obstruct the common bile duct. Which of the following clinical manifestations would indicate a common bile duct obstruction associated with a tumor in the head of the pancreas? Choose all that apply.

Trauma, such as penetrating wounds or compound fractures Vascular insufficiency in clients with diabetes or peripheral vascular disease Surgical contamination, such as pin sites of skeletal traction

The client has just been diagnosed with osteomyelitis. What are possible causes of osteomyelitis? Select all that apply.

Place the client in the Fowler's position.

The client has returned to the floor following a radical neck dissection. Anesthesia has worn off. What is the nurse's priority action?

shift

The client is on a continuous tube feeding. The nurse determines the tube placement should be checked every

Elastic compression bandages

The health care provider is preparing to bivalve the client's cast. Which supplies should the nurse assemble?

C) Flat bones

The human body is designed to protect its vital parts. A fracture of what type of bone may interfere with the protection of vital organs? A) Long bones B) Short bones C) Flat bones D) Irregular bones

the increased potential for aspiration.

The most significant complication related to continuous tube feedings is

Place the client in semi-Fowler's position.

The nurse cares for a client with cholecystitis with severe biliary colic symptoms. Which nursing intervention best promotes adequate respirations in a client with these symptoms?

continuous

The nurse collaborates with the physician and dietician to determine the best type of tube feeding for a client at risk for diarrhea due to hypertonic feeding solutions. Which type of feedings should the nurse suggest?

joint stiffness decreased range of motion decreased endurance

The nurse is admitting an older adult to a skilled nursing facility. What assessment parameters will the nurse expect to find with the musculoskeletal assessment? Select all that apply.

Severe mid-abdominal to upper abdominal pain radiating to both sides and to the back

The nurse is asking the client with acute pancreatitis to describe the pain. What pain symptoms does the client describe related to acute pancreatitis?

Alterations in mood Agitation Insomnia

The nurse is assessing a client with hepatic cirrhosis for mental deterioration. For what clinical manifestations will the nurse monitor? Select all that apply.

AC

The nurse is assessing a patient for dietary factors that may influence her risk for osteoporosis. The nurse should question the patient about her intake of what nutrients? Select all that apply. A) Calcium B) Simple carbohydrates C) Vitamin D D) Protein E) Soluble fiber

C) Leave the tube in its present position.

The nurse is assessing placement of a nasogastric tube that the patient has had in place for 2 days. The tube is draining green aspirate. What is the nurses most appropriate action? A) Inform the physician that the tube may be in the patients pleural space. B) Withdraw the tube 2 to 4 cm. C) Leave the tube in its present position. D) Advance the tube up to 8 cm

2 in.

The nurse is assisting a client to drain his continent ileostomy (Kock pouch). The nurse should insert the catheter how far through the nipple/valve?

Fecal incontinence

The nurse is caring for a client with a suspected megacolon. The nurse anticipates that one of the findings of assessment will be

Measure abdominal girth daily. Perform daily weights.

The nurse is caring for a patient who has ascites as a result of hepatic dysfunction. What intervention can the nurse provide to determine if the ascites is increasing? (Select all that apply.)

B) Bleeding and perforation

The nurse is caring for a patient who has just returned from the ERCP removal of gallstones. The nurse should monitor the patient for signs of what complications? A) Pain and peritonitis B) Bleeding and perforation C) Acidosis and hypoglycemia D) Gangrene of the gallbladder and hyperglycemia

B) Foul-smelling diarrhea that contains fat

The nurse is caring for a patient who is undergoing diagnostic testing for suspected malabsorption. When taking this patients health history and performing the physical assessment, the nurse should recognize what finding as most consistent with this diagnosis? A) Recurrent constipation coupled with weight loss B) Foul-smelling diarrhea that contains fat C) Fever accompanied by a rigid, tender abdomen D) Bloody bowel movements accompanied by fecal incontinence

C) Protect the affected leg from internal rotation.

The nurse is caring for a patient who underwent a total hip replacement yesterday. What should the nurse do to prevent dislocation of the new prosthesis? A) Keep the affected leg in a position of adduction. B) Have the patient reposition himself independently. C) Protect the affected leg from internal rotation. D) Keep the hip flexed by placing pillows under the patients knee.

A) Compare parts of the body symmetrically.

The nurse is performing an assessment of a patients musculoskeletal system and is appraising the patients bone integrity. What action should the nurse perform during this phase of assessment? A) Compare parts of the body symmetrically. B) Assess extremities when in motion rather than at rest. C) Percuss as many joints as are accessible. D) Administer analgesia 30 to 60 minutes before assessment.

Risk for injury related to altered clotting mechanisms Activity intolerance related to fatigue, general debility, muscle wasting, and discomfort Disturbed body image related to changes in appearance, sexual dysfunction, and role function

The nurse is preparing a care plan for a client with hepatic cirrhosis. Which nursing diagnoses are appropriate? Select all that apply.

"Are you wearing any jewelry?" "Have you removed your hearing aid?" "Do you have a pacemaker?"

The nurse is preparing the client with a right neck mass for magnetic resonance imaging (MRI). Which question should the nurse ask? Select all that apply.

Explain the risks of flexion contracture to the client.

The nurse is providing care for a client who has had a below-the-knee amputation. The nurse enters the client's room and finds the client resting in bed with his residual limb supported on pillow. What is the nurse's most appropriate action?

Smoking history of 20 years Male gender Previous treatment for gastroesophageal reflux disease

The nurse is reviewing the chart of a client with swallowing problems. Which factors would raise suspicion that the client has cancer of the esophagus? Select all that apply.

Pancakes with butter and honey, and orange juice

The nurse is teaching a client who was admitted to the hospital with acute hepatic encephalopathy and ascites about an appropriate diet. The nurse determines that the teaching has been effective when the client chooses which food choice from the menu?

Sialadenitis

The nurse notes that a client has inflammation of the salivary glands. The nurse documents which finding?

Do not eat and drink at the same time. Drink plenty of water, from 90 minutes after each meal to 15 minutes before each meal. Avoid fruit drinks and soda.

The nurse reviews dietary guidelines with a client who had a gastric banding. Which teaching points are included? Select all that apply.

C) Lordosis

The results of a nurses musculoskeletal examination show an increase in the lumbar curvature of the spine. The nurse should recognize the presence of what health problem? A) Osteoporosis B) Kyphosis C) Lordosis D) Scoliosis

A) Organic fruit juice

The school nurse is planning a health fair for a group of fifth graders and dental health is one topic that the nurse plans to address. What would be most likely to increase the risk of tooth decay? A) Organic fruit juice B) Roasted nuts C) Red meat that is high in fat D) Cheddar cheese

Shortened, adducted, and externally rotated

What assessment findings of the leg are consistent with a fracture of the femoral neck?

Weight loss

What is a major concern for the nurse when caring for a patient with chronic pancreatitis?

Educate the client on cast care and complications

What is the best action by the nurse to achieve optimal outcomes when caring for a client with a musculoskeletal disorder who is using a cast?

sudden, sustained abdominal pain abdominal distention

What symptoms of perforation might the nurse observe in a client with an intestinal obstruction? Select all that apply.

change in mental status signs of GI bleeding

When caring for a client with cirrhosis, which symptoms should a nurse report immediately? Select all that apply.

acetaminophen

_____has been identified as the leading cause of acute liver failure

avulsion

a fracture in which a fragment of bone has been pulled away by a tendon and its attachment

communited

a fracture in which bone has splintered into several fragments

green stick

a fracture in which one side of a bone is broken and the other side is bent

oblique

a fracture occurring at an angle across the bone

scoliosis

a lateral curving deviation of the spine

non-viral hepatitis

certain chemicals have toxic effects on the liver and produce acute liver cell necrosis and toxic hepatitis when inhaled, injected parenerally, or taken by mouth. The chemicals most commonly implicated in this disease are carbon tetrachloride, phosphorus, chloroform, and gold compounds

stage 4 hepatic encephalopathy

comatose, may not respond to painful stimuli

knock knees

genu valgum

bow legged

genu varum

peroneal nerve damage

if a patient experiences foot drop what nerve is damaged

volkmann's ischemic contracture

impairment of motor function and Sensibility, contracture of the fingers and wrist. Contracture the fingers and wrist occurs related to obstructed arterial blood flow

biliary cirrhosis

in which scarring occurs in the liver around the bile ducts. This type of cirrhosis usually results from chronic biliary obstruction and infection it is much less common

alcoholic cirrhosis

in which the scar tissue characteristically surrounds the portal areas. This is most frequently caused by chronic alcoholism and is the most common type of cirrhosis

postnecrotic cirrhosis

in which there are broad bands of scar tissue. This is a late result of previous bout of acute viral hepatitis

constructional apraxia

inability to reproduce a simple figure in two or three dimensions is referred to as what

stage 2 hepatic encephalopathy

increase drowsiness, disorientation, inappropriate behavior, mood swings, agitation

osteomyelitis

infection of the bone is known as what

strain

injury to a muscle or tendon from overuse, overstretching, or excessive stress may cause what

kyphosis

is an increased forward curvature of the thoracic spine that causes a Boeing or rounding of the back leading to Hunchback or slouching posture

acetylcholine

is released from vesicles and stimulates muscles to contract

fulminant hepatic failure

is the clinical syndrome of sudden and severely impaired liver function in a previously healthy person

GGT

levels are associated with cholestasis; alcoholic liver disease

ALT alanine aminotransferase

levels increase primarily in liver disorders, used to monitor the course of hepatitis, cirrhosis, the effects of tx that may be toxic to the liver

herb milk thistle

many patients with end-stage liver disease with cirrhosis use what to treat jaundice and other symptoms

osteoporosis

most common bone disorder in the western world. second to arthritis as a cause of musculoskeletal morbidity in the elderly

stage 1 of hepatic encephalopathy

normal level of Consciousness with periods of lethargy and Euphoria reversal day night sleep patterns

a third-degree sprain

occurs when a ligament is completely torn or ruptured. May also cause an avulsion of the bone. Symptoms include severe pain, tenderness, increase the Dima, and abnormal joint motion

acute pancreatitis

pancreatic duct becomes obstructed and enzymes back up, causing autodigestion and inflammation of the pancreas

arthrodesis

perform to relieve pain and to restore stability and Alignment. Results in loss motion. Also known as joint Fusion

acute pancreatitis

presents with fever, jaundice, confusion, agitation, ecchymosis in the flank or umbilical area, and abdominal guarding

tenesmus

refers to ineffectual straining at stool. Azotorrhea refers to excess of nitrogenous matter in the feces or urine

diverticulitits

refers to inflammation of a diverticulum from obstruction (by fecal matter) resulting in abscess formation.

Obstructive Jaundice

resulting from extra hepatic obstruction may be caused by occlusion of the bile duct from a gallstone and inflammatory process, a tumor, or pressure from an enlarged organ dark organge-brown urine, steattorhea dyspepsia and intolerance of fats, impaired digestion pruritus can not flow normally into intestines and backs up into the liver

stage 3 hepatic encephalopathy

stuporous, difficult to Rouse, sleeps most of the time, Marked confusion, incoherent speech

lordosis

swayback, an exaggerated curvature of the lumbar spine

true

true or False, only persons with hepatitis B are at risk for hepatitis D

false

true or false, bleeding esophageal varices result in an increase in renal perfusion

toxic hepatitis and drug-induced hepatitis

two forms of non-viral hepatitis are

mental changes and motor disturbances

what are early signs of hepatic encephalopathy

accumulation of ammonia and other toxic metabolites in the blood

what are life-threatening complications of hepatic encephalopathy

vitamin D and calcium

what do you need adequate amounts of if diagnosed with osteoporosis

recurrent attacks of severe upper abdominal and back pain accompanied by vomiting

what is a major symptom of chronic pancreatitis

compartment syndrome

what is the most serious complication of casting and splinting

vasopressin, pitressin

what medication may be the initial mode of therapy in urgent situations because it produces constriction of the splanichnic arteriole bed and decreases portal pressure

spironolactone; aldactone

which diuretic medication would most often be used for a patient with ascites?

Hemorrhoids

A client asks the nursing assistant for a bedpan. When the client is finished, the nursing assistant notifies the nurse that the client has bright red streaking of blood in the stool. The nurse's assessment should focus on what potential cause?

Measure and record drainage.

A client has a gastrostomy tube that has been placed to drain stomach contents by low intermittent suction. What is the nurse's priority during this aspect of the client's care?

Compares exposed tube length with original measurement Visually assesses the color of the aspirate Checks the pH of the gastric contents

A client has a nasogastric tube for continuous tube feeding. The nurse does all the following every shift to verify placement (select all options that apply):

Keep the vent lumen above the client's waist.

A client is concerned about leakage of gastric contents out of the gastric sump tube the nurse has just inserted. What would the nurse do to prevent reflux gastric contents from coming through the blue vent of a gastric sump tube?

No land line; cell phone available and taken by family member during working hours

A client is receiving parenteral nutrition (PN) through a peripherally inserted central catheter (PICC) and will be discharged home with PN. The home health nurse evaluates the home setting and would make a recommendation when noticing which circumstance?

D) Brushing the patients teeth with a toothbrush and small amount of toothpaste

A nurse is providing oral care to a patient who is comatose. What action best addresses the patients risk of tooth decay and plaque accumulation? A) Irrigating the mouth using a syringe filled with a bacteriocidal mouthwash B) Applying a water-soluble gel to the teeth and gums C) Wiping the teeth and gums clean with a gauze pad D) Brushing the patients teeth with a toothbrush and small amount of toothpaste

D) An internal retention disc secures the tube against the stomach wall.

A patient with dysphagia is scheduled for PEG tube insertion and asks the nurse how the tube will stay in place. What is the nurses best response? A) Adhesive holds a flange in place against the abdominal skin. B) A stitch holds the tube in place externally. C) The tube is stitched to the abdominal skin externally and the stomach wall internally. D) An internal retention disc secures the tube against the stomach wall.

C) Fluids must be increased to facilitate the evacuation of the stool.

An adult patient is scheduled for an upper GI series that will use a barium swallow. What teaching should the nurse include when the patient has completed the test? A) Stool will be yellow for the first 24 hours postprocedure. B) The barium may cause diarrhea for the next 24 hours. C) Fluids must be increased to facilitate the evacuation of the stool. D) Slight anal bleeding may be noted as the barium is passed.

D) Streaks of blood present in the stool

An inpatient has returned to the medical unit after a barium enema. When assessing the patients subsequent bowel patterns and stools, what finding should the nurse report to the physician? A) Large, wide stools B) Milky white stools C) Three stools during an 8-hour period of time D) Streaks of blood present in the stool

D) Avoid vitamin C for 72 hours before you start the test.

Probably the most widely used in-office or at-home occult blood test is the Hemoccult II. The patient has come to the clinic because he thinks there is blood in his stool. When you reviewed his medications, you noted he is on antihypertensive drugs and NSAIDs for early arthritic pain. You are sending the patient home with the supplies necessary to perform 2 hemoccult tests on his stool and mail the samples back to the clinic. What instruction would you give this patient? A) Take all your medications as usual. B) Take all your medications except the antihypertensive medications. C) Dont eat highly acidic foods 72 hours before you start the test. D) Avoid vitamin C for 72 hours before you start the test.

Weigh the client every day. Check blood glucose level every 6 hours. Document intake and output.

The client cannot tolerate oral feedings due to an intestinal obstruction and is NPO. A central line has been inserted, and the client is being started on parenteral nutrition (PN). What actions would the nurse perform while the client receives PN? Select all that apply.

C) I flush my tube with water before and after each of my medications.

The management of the patients gastrostomy is an assessment priority for the home care nurse. What statement would indicate that the patient is managing the tube correctly? A) I clean my stoma twice a day with alcohol. B) The only time I flush my tube is when Im putting in medications. C) I flush my tube with water before and after each of my medications. D) I try to stay still most of the time to avoid dislodging my tube.

Until bowel sound is present Until flatus is passed Until peristalsis is resumed

The nurse is inserting a nasoenteric tube for a patient with a paralytic ileus. How long does the nurse anticipate the tube will be required? (Select all that apply.)

Serum antibodies for H. pylori

The nurse is instructing the client who was newly diagnosed with peptic ulcers. Which of the following diagnostic studies would the nurse anticipate reviewing with the client?

B) The patient is agitated.

The nurse is preparing to insert a patients ordered NG tube. What factor should the nurse recognize as a risk for incorrect placement? A) The patient is obese and has a short neck. B) The patient is agitated. C) The patient has a history of gastroesophageal reflux disease (GERD). D) The patient is being treated for pneumonia.

C) 2 to 3 months

You are caring for a patient who was admitted to have a low-profile gastrostomy device (LPGD) placed. How soon after the original gastrostomy tube placement can an LPGD be placed? A) 2 weeks B) 4 to 6 weeks C) 2 to 3 months D) 4 to 6 months

Intrinsic factor

A client is diagnosed with megaloblastic anemia caused by vitamin B12 deficiency. The health care provider begins the client on cyanocobalamin (Betalin-12), 100 mcg I.M. daily. Which substance influences vitamin B12 absorption?

Change the dressing no more than weekly.

A nurse is aware of the high incidence of catheter-related bloodstream infections in clients receiving parenteral nutrition. What nursing action has the greatest potential to reduce catheter-related bloodstream infections?

Clotted or displaced catheter Pneumothorax Hyperglycemia Line sepsis

A nurse is creating a care plan for a client who is receiving parenteral nutrition. The client's care plan should include nursing actions relevant to what potential complications? Select all that apply.

Masks Skin antiseptic Alcohol wipes Sterile gauze pads

A nurse is preparing to perform a dressing change to the site of a client's central venous catheter used for parenteral nutrition. Which equipment and supplies would the nurse need to gather? Select all that apply.

Shakiness Tachycardia Weakness Confusion

A nurse suspects that a patient is developing rebound hypoglycemia secondary to parenteral nutrition being discontinued too rapidly. Which of the following would support the nurse's suspicions? Select all that apply.

D) Inform the primary care provider of this finding.

The nurse is caring for a patient who has a diagnosis of AIDS. Inspection of the patients mouth reveals the new presence of white lesions on the patients oral mucosa. What is the nurses most appropriate response? A) Encourage the patient to gargle with salt water twice daily. B) Attempt to remove the lesions with a tongue depressor. C) Make a referral to the units dietitian. D) Inform the primary care provider of this finding.

C) Gently rotate the tube.

The nurse is caring for a patient who is postoperative from having a gastrostomy tube placed. What should the nurse do on a daily basis to prevent skin breakdown? A) Verify tube placement. B) Loop adhesive tape around the tube and connect it securely to the abdomen. C) Gently rotate the tube. D) Change the wet-to-dry dressing.

CDE

The nurse is caring for a patient with a duodenal ulcer and is relating the patients symptoms to the physiologic functions of the small intestine. What do these functions include? Select all that apply. A) Secretion of hydrochloric acid (HCl) B) Reabsorption of water C) Secretion of mucus D) Absorption of nutrients E) Movement of nutrients into the bloodstream

Flush with 10 mL of water.

The nurse is to discontinue a nasogastric tube that had been used for decompression. What is the first action the nurse should take?

Provide frequent mouth care.

The nurse observes dry mucous membranes in a client who is receiving tube feedings after an oral surgery. The client also reports unpleasant tastes and odors. Which measure should be included in the client's plan of care?

The client must fast for 8 hours before the examination. The throat will be sprayed with a local anesthetic. After gastroscopy, the client cannot eat or drink until the gag reflex returns (1 to 2 hours).

The nurse is providing instructions to a client scheduled for a gastroscopy. What should the nurse be sure to include in the instructions? Select all that apply.

The client will change positions frequently throughout the procedure.

The nurse is teaching the client about the upcoming endoscopic retrograde cholangiopancreatography (ERCP). Although the nurse instructs on several pertinent points of care, which is emphasized?

Rovsing sign

The nurse observes the physician palpating the abdomen of a client that is suspected of having acute appendicitis. When the abdomen is pressed in the left lower quadrant the client complains of pain on the right side. What does the nurse understand this assessment technique is referred to?

Serum lipase

The nurse should assess for an important early indicator of acute pancreatitis. What prolonged and elevated level would the nurse determine is an early indicator?

Humerus

The orthopedic nurse should assess for signs and symptoms of Volkmann contracture if a client has fractured which of the following bones?

C) The absorption into the bloodstream of nutrient molecules produced by digestion

The physiology instructor is discussing the GI system with the pre-nursing class. What should the instructor describe as a major function of the GI tract? A) The breakdown of food particles into cell form for digestion B) The maintenance of fluid and acid-base balance C) The absorption into the bloodstream of nutrient molecules produced by digestion D) The control of absorption and elimination of electrolytes

Catheter hub

The primary source of microorganisms for catheter-related infections are the skin and which of the following?

Placing a trapeze on the bed Ensuring that the weights are hanging freely Assessing the client's alignment in the bed Frequently assessing pain level

Which actions by the nurse demonstrate an understanding of caring for a client in traction? Select all that apply.

Osteomalacia

Which condition is a metabolic bone disease characterized by inadequate mineralization of bone?

Hemorrhage

Which factor may contribute to compartment syndrome?

Chronic irritation of the esophagus is a known risk factor.

Which is an accurate statement regarding cancer of the esophagus?

Nonunion

Which of the following describes failure of the ends of a fractured bone to unite in normal alignment?

Pepsin

Which of the following is an enzyme secreted by the gastric mucosa?

Compression

Which of the following type of fracture is associated with osteoporosis?

Pneumonia Skin breakdown Sepsis Delirium

A 75-year-old client had surgery for a left hip fracture yesterday. When completing the plan of care, the nurse should include assessment for which complications? Select all that apply.

Metastases are common and respond poorly to treatment.

A client has been diagnosed with a malignancy of the oral cavity and is undergoing oncologic treatment. The oncologic nurse is aware that the prognosis for recovery from head and neck cancers is often poor because of what characteristic of these malignancies?

NSAIDs activity modification and joint rest arthroscopic surgery

A client has been diagnosed with a rotator cuff tear. What are the options for treating this condition? Select all that apply.

Regurgitation of undigested food

A client has been diagnosed with an esophageal diverticulum after undergoing diagnostic imaging. When taking the health history, the nurse should expect the client to describe what sign or symptom?

Administer oxygen.

A client is hospitalized for open reduction of a fractured femur. During the postoperative assessment, the nurse notes that the client is restless and observes petechiae on the client's chest. Which nursing action is indicated first?

floor of the mouth

A client is in the initial stages of oral cancer diagnosis and is frightened about the side effects of treatment and subsequent prognosis. The client has many questions regarding this type of cancer and asks where oral cancer typically occurs. What is the nurse's response?

Daily weights and abdominal girth measurement

A client with portal hypertension has been admitted to the medical floor. The nurse should prioritize what assessments?

Enhancement of verbal communication

A nurse is caring for a client who has had surgery for oral cancer. When addressing the client's long-term needs, the nurse should prioritize interventions and referrals with what goal?

peptic ulcer disease

A nurse is caring for a client who is undergoing a diagnostic workup for a suspected gastrointestinal problem. The client reports gnawing epigastric pain following meals and heartburn. What would the nurse suspect this client has?

An elevated serum alkaline phosphatase level and a normal serum calcium level

A nurse is caring for a client with Paget disease and is reviewing the client's most recent laboratory values. Which of the following values is most characteristic of Paget disease?

Support the affected extremity with external supports such as splints.

A nurse is caring for a client with a bone tumor. The nurse is providing education to help the client reduce the risk for pathologic fractures. What should the nurse teach the client?

BCD

A nurse is caring for a patient admitted with a suspected malabsorption disorder. The nurse knows that one of the accessory organs of the digestive system is the pancreas. What digestive enzymes does the pancreas secrete? Select all that apply. A) Pepsin B) Lipase C) Amylase D) Trypsin E) Ptyalin

A,c,e

A nurse is caring for a patient in the late stages of esophageal cancer. The nurse should plan to prevent or address what characteristics of this stage of the disease? Select all that apply. A) Perforation into the mediastinum B) Development of an esophageal lesion C) Erosion into the great vessels D) Painful swallowing E) Obstruction of the esophagus

C) Alkaline phosphatase

A nurse is caring for a patient with a diagnosis of cancer that has metastasized. What laboratory value would the nurse expect to be elevated in this patient? A) Bilirubin B) Potassium C) Alkaline phosphatase D) Creatinine

D) Change in bowel habits

A nurse is teaching a group of adults about screening and prevention of colorectal cancer. The nurse should describe which of the following as the most common sign of possible colon cancer? A) Development of new hemorrhoids B) Abdominal bloating and flank pain C) Unexplained weight gain D) Change in bowel habits

A) Pricking the skin between the great and second toe

A nurse on the orthopedic unit is assessing a patients peroneal nerve. The nurse will perform this assessment by doing which of the following actions? A) Pricking the skin between the great and second toe B) Stroking the skin on the sole of the patients foot C) Pinching the skin between the thumb and index finger D) Stroking the distal fat pad of the small finger

Proton pump inhibitors.

A nurse practitioner, who is treating a patient with GERD, knows that responsiveness to this drug classification is validation of the disease. The drug classification is:

A) Measure and record drainage.

A patient has a gastrostomy tube that has been placed to drain stomach contents by low intermittent suction. What is the nurses priority during this aspect of the patients care? A) Measure and record drainage. B) Monitor drainage for change in color. C) Titrate the suction every hour. D) Feed the patient via the G tube as ordered.

B) Application of heat 15 to 20 minutes each hour

A patient has had a laparoscopic cholecystectomy. The patient is now complaining of right shoulder pain. What should the nurse suggest to relieve the pain? A) Aspirin every 4 to 6 hours as ordered B) Application of heat 15 to 20 minutes each hour C) Application of an ice pack for no more than 15 minutes D) Application of liniment rub to affected area

B) Apply a skin barrier to the peristomal skin prior to applying the pouch.

A patient has had an ileostomy created for the treatment of irritable bowel disease and the patient is now preparing for discharge. What should the patient be taught about changing this device in the home setting? A) Apply antibiotic ointment as ordered after cleaning the stoma. B) Apply a skin barrier to the peristomal skin prior to applying the pouch. C) Dispose of the clamp with each bag change. D) Cleanse the area surrounding the stoma with alcohol or chlorhexidine.

Transjugular intrahepatic portosystemic shunting (TIPS)

A patient with bleeding esophageal varices has had pharmacologic therapy with Octreotide (Sandostatin) and endoscopic therapy with esophageal varices banding, but the patient has continued to have bleeding. What procedure that will lower portal pressure does the nurse prepare the patient for?

low platelet count.

A physician has ordered a liver biopsy for a client with cirrhosis whose condition has recently deteriorated. The nurse reviews the client's recent laboratory findings and recognizes that the client is at risk for complications due to:

Wide resection of the middle and distal portions of the stomach with removal of about 75% of the stomach

After a client received a diagnosis of gastric cancer, the surgical team decides that a Billroth II would be the best approach to treatment. How would the nurse explain this procedure to the family?

A) Following proper hand-washing techniques

During a health education session, a participant has asked about the hepatitis E virus. What prevention measure should the nurse recommend for preventing infection with this virus? A) Following proper hand-washing techniques B) Avoiding chemicals that are toxic to the liver C) Wearing a condom during sexual contact D) Limiting alcohol intake

B) Increased fluid and fiber intake

During a patients scheduled home visit, an older adult patient has stated to the community health nurse that she has been experiencing hemorrhoids of increasing severity in recent months. The nurse should recommend which of the following? A) Regular application of an OTC antibiotic ointment B) Increased fluid and fiber intake C) Daily use of OTC glycerin suppositories D) Use of an NSAID to reduce inflammation

The client's hands flap back and forth when the arms are extended.

The nurse is completing a morning assessment of a client with cirrhosis. Which information obtained by the nurse will be of most concern?

Peroneal

The nurse is conducting a musculoskeletal assessment of a client in a nursing home. The client is unable to dorsiflex the right foot or extend the toes. The nurse evaluates this finding as an injury to which nerve?

"We need an adequate amount of exposure to sunshine."

The nurse is educating a group of women on the prevention of osteoporosis. The nurse recognizes the education as being effective when the group members make which statement?

sprain

an injury to the ligaments and tendons that surround a joint. It is caused by a twisting motion or hyperextension of a joint

hepatocellular jaundice

caused by the inability of damaged liver cells to clear normal amounts of bili from the blood Mild or severlly ill lack of appetite/N+V/weight loss Tired/weakness Flu like symptoms, headache, chills, fever infection

borborygmus

is the intestinal rumbling caused by the movement of gas through the intestines that accompanies diarrhea

nausea and vomiting, stomach pain, jaundice, dark urine, Galleria, joint pain, dyspepsia, decreased appetite, abdominal pain, malaise

signs and symptoms of hepatitis are

Electrical stimulation

A client has delayed bone healing in a fractured right humerus. What should the nurse prepare the client for that promotes bone growth?

An absence of blood in stool

A client's health history is suggestive of inflammatory bowel disease. Which of the following would suggest Crohn's disease, rather than ulcerative colitis, as the cause of the client's signs and symptoms?

C) Increased uric acid levels

A clinic nurse is caring for a patient with suspected gout. While explaining the pathophysiology of gout to the patient, the nurse should describe which of the following? A) Autoimmune processes in the joints B) Chronic metabolic acidosis C) Increased uric acid levels D) Unstable serum calcium levels

C) Inability to take in adequate oral food or fluids within 7 days

A critical care nurse is caring for a patient diagnosed with acute pancreatitis. The nurse knows that the indications for starting parenteral nutrition (PN) for this patient are what? A) 5% deficit in body weight compared to preillness weight and increased caloric need B) Calorie deficit and muscle wasting combined with low electrolyte levels C) Inability to take in adequate oral food or fluids within 7 days D) Significant risk of aspiration coupled with decreased level of consciousness

D) Right shoulder

A nurse is assessing a patient who has been diagnosed with cholecystitis, and is experiencing localized abdominal pain. When assessing the characteristics of the patients pain, the nurse should anticipate that it may radiate to what region? A) Left upper chest B) Inguinal region C) Neck or jaw D) Right shoulder

D) Notify the physician.

A nurse is caring for a patient who has a diagnosis of GI bleed. During shift assessment, the nurse finds the patient to betachycardic and hypotensive, and the patient has an episode of hematemesis while the nurse is in the room. In addition to monitoring the patients vital signs and level of conscious, what would be a priority nursing action for this patient? A) Place the patient in a prone position. B) Provide the patient with ice water to slow any GI bleeding. C) Prepare for the insertion of an NG tube. D) Notify the physician.

ACE

A nurse is caring for a patient who has a gastrointestinal tube in place. Which of the following are indications for gastrointestinal intubation? Select all that apply. A) To remove gas from the stomach B) To administer clotting factors to treat a GI bleed C) To remove toxins from the stomach D) To open sphincters that are closed E) To diagnose GI motility disorders

D) Assess the pin insertion site every 8 hours.

A nurse is caring for a patient who is in skeletal traction. To prevent the complication of skin breakdown in a patient with skeletal traction, what action should be included in the plan of care? A) Apply occlusive dressings to the pin sites. B) Encourage the patient to push up with the elbows when repositioning. C) Encourage the patient to perform isometric exercises once a shift. D) Assess the pin insertion site every 8 hours.

C) Signs of neurovascular compromise

An elderly patients hip joint is immobilized prior to surgery to correct a femoral head fracture. What is the nurses priority assessment? A) The presence of leg shortening B) The patients complaints of pain C) Signs of neurovascular compromise D) The presence of internal or external rotation

B) Below the right nipple

A nurse is caring for a patient with biliary colic and is aware that the patient may experience referred abdominal pain. Where would the nurse most likely expect this patient to experience referred pain? A) Midline near the umbilicus B) Below the right nipple C) Left groin area D) Right lower abdominal quadrant

D) Apply local anesthetic to the back of the patients throat.

A nurse is caring for a patient with recurrent hematemesis who is scheduled for upper gastrointestinal fibroscopy (UGF). How should the nurse in the radiology department prepare this patient? A) Insert a nasogastric tube. B) Administer a micro Fleet enema at least 3 hours before the procedure. C) Have the patient lie in a supine position for the procedure. D) Apply local anesthetic to the back of the patients throat.

D) Use a combination of at least two accepted methods for confirming placement. measurement of the tube auscultation xray

A nurse is creating a care plan for a patient with a nasogastric tube. How should the nurse direct other members of the care team to check correct placement of the tube? A) Auscultate the patients abdomen after injecting air through the tube. B) Assess the color and pH of aspirate. C) Locate the marking made after the initial x-ray confirming placement. D) Use a combination of at least two accepted methods for confirming placement.

Sit the client in an upright position Apply gloves to the nurse's hands Measure the length of the tube that will be inserted Apply water-soluble lubricant to the tip of the tube Tilt the client's nose upward Instruct the client to lower the head and swallow

A nurse is inserting a nasogastric tube for feeding a client. Place in order the steps from 1 to 6 for correctly inserting the tube.

D) IV administration of octreotide (Sandostatin)

A nurse is participating in the emergency care of a patient who has just developed variceal bleeding. What intervention should the nurse anticipate? A) Infusion of intravenous heparin B) IV administration of albumin C) STAT administration of vitamin K by the intramuscular route D) IV administration of octreotide (Sandostatin)

D) Crepitus

A nurse is performing a musculoskeletal assessment of a patient with arthritis. During passive range-of-motion exercises, the nurse hears an audible grating sound. The nurse should document the presence of which of the following? A) Fasciculations B) Clonus C) Effusion D) Crepitus

A) Examine ones own attitudes towards obesity in general and the patient in particular.

A nurse is performing the admission assessment of a patient whose high body mass index (BMI) corresponds to class III obesity. In order to ensure empathic and patient-centered care, the nurse should do which of the following? A) Examine ones own attitudes towards obesity in general and the patient in particular. B) Dialogue with the patient about the lifestyle and psychosocial factors that resulted in obesity. C) Describe ones own struggles with weight gain and weight loss to the patient. D) Elicit the patients short-term and long-term goals for weight loss.

"It can evaluate the presence and location of ductal stones and aid in stone removal." "It can assess the anatomy of the pancreas and the pancreatic and biliary ducts." "It can detect unhealthy tissues in the pancreas and assess for abscesses and pseudocysts."

A nurse is preparing a client for endoscopic retrograde cholangiopancreatography (ERCP). The client asks what this test is used for. Which statements by the nurse explains how ERCP can determine the difference between pancreatitis and other biliary disorders? Select all that apply.

B) Impaired Skin Integrity

A nurse is preparing a plan of care for a patient with pancreatic cysts that have necessitated drainage through the abdominal wall. What nursing diagnosis should the nurse prioritize? A) Disturbed Body Image B) Impaired Skin Integrity C) Nausea D) Risk for Deficient Fluid Volume

A) Intravenous fat emulsions may be infused simultaneously with PN through a Y-connector close to the infusion site and should not be filtered.

A nurse is preparing to administer a patients intravenous fat emulsion simultaneously with parenteral nutrition (PN). Which of the following principles should guide the nurses action? A) Intravenous fat emulsions may be infused simultaneously with PN through a Y-connector close to the infusion site and should not be filtered. B) The nurse should prepare for placement of another intravenous line, as intravenous fat emulsions may not be infused simultaneously through the line used for PN. C) Intravenous fat emulsions may be infused simultaneously with PN through a Y-connector close to the infusion site after running the emulsion through a filter. D) The intravenous fat emulsions can be piggy-backed into any existing IV solution that is infusing.

D) Antibiotics, proton pump inhibitors, and bismuth salts

32. A patient has been diagnosed with peptic ulcer disease and the nurse is reviewing his prescribed medication regimen with him. What is currently the most commonly used drug regimen for peptic ulcers? A) Bismuth salts, antivirals, and histamine-2 (H2) antagonists B) H2 antagonists, antibiotics, and bicarbonate salts C) Bicarbonate salts, antibiotics, and ZES D) Antibiotics, proton pump inhibitors, and bismuth salts

Stress on the weakened bone must be avoided.

A nurse is collaborating with the physical therapist to plan the care of a client with osteomyelitis. What principle should guide the management of activity and mobility in this client?

A) Hydrochloric acid is secreted by glands in the stomach in response to the actual or anticipated presence of food.

A patient presents at the walk-in clinic complaining of recurrent sharp stomach pain that is relieved by eating. The nurse suspects that the patient may have an ulcer. How would the nurse explain the formation and role of acid in the stomach to the patient? A) Hydrochloric acid is secreted by glands in the stomach in response to the actual or anticipated presence of food. B) As digestion occurs in the stomach, the stomach combines free hydrogen ions from the food to form acid. C) The body requires an acidic environment in order to synthesize pancreatic digestive enzymes; the stomach provides this environment. D) The acidic environment in the stomach exists to buffer the highly alkaline environment in the esophagus.

A) Make sure you dont bring your knees close together.

A patient was brought to the emergency department after a fall. The patient is taken to the operating room to receive a right hip prosthesis. In the immediate postoperative period, what health education should the nurse emphasize? A) Make sure you dont bring your knees close together. B) Try to lie as still as possible for the first few days. C) Try to avoid bending your knees until next week. D) Keep your legs higher than your chest whenever you can.

Hairy leukoplakia

A patient who is HIV positive comes to the clinic and is experiencing white patches with rough hairlike projections. The nurse observes the lesions on the lateral border of the tongue. What abnormality of the mouth does the nurse determine these lesions are?

C) Regurgitation and aspiration are less likely.

A patient who suffered a stroke had an NG tube inserted to facilitate feeding shortly after admission. The patient has since become comatose and the patients family asks the nurse why the physician is recommending the removal of the patients NG tube and the insertion of a gastrostomy tube. What is the nurses best response? A) It eliminates the risk for infection. B) Feeds can be infused at a faster rate. C) Regurgitation and aspiration are less likely. D) It allows caregivers to provide personal hygiene more easily.

C) Monitor the patient closely for further signs of dumping syndrome.

A patient who underwent gastric banding 3 days ago is having her diet progressed on a daily basis. Following her latest meal, the patient complains of dizziness and palpitations. Inspection reveals that the patient is diaphoretic. What is the nurses best action? A) Insert a nasogastric tube promptly. B) Reposition the patient supine. C) Monitor the patient closely for further signs of dumping syndrome. D) Assess the patient for signs and symptoms of aspiration.

A) Rheumatoid arthritis (RA)

A patients decreased mobility is ultimately the result of an autoimmune reaction originating in the synovial tissue, which caused the formation of pannus. This patient has been diagnosed with what health problem? A) Rheumatoid arthritis (RA) B) Systemic lupus erythematosus C) Osteoporosis D) Polymyositis

B) Arthritis

A public health nurse is organizing a campaign that will address the leading cause of musculoskeletal-related disability in the United States. The nurse should focus on what health problem? A) Osteoporosis B) Arthritis C) Hip fractures D) Lower back pain

C) Blood glucose levels

A student nurse is caring for a patient who has a diagnosis of acute pancreatitis and who is receiving parenteral nutrition. The student should prioritize which of the following assessments? A) Fluid output B) Oral intake C) Blood glucose levels D) BUN and creatinine levels

Abdominal surgery

After teaching a group of students about intestinal obstruction, the instructor determines that the teaching was effective when the students identify which of the following as a cause of a functional obstruction?

Promoting weight-bearing exercises

An older adult client with a diagnosis of left-sided stroke is admitted to the facility. To prevent the development of disuse osteoporosis, which objective is most appropriate?

Evaluate the client's understanding of the procedure.

As a nurse completes the admission assessment of a client admitted for gastric bypass surgery, the client states, "Finally! I'll be thin and able to eat without much concern." How should the nurse intervene?

C) Frequent lung auscultation

A nurse is providing care for a patient with a diagnosis of late-stage Alzheimers disease. The patient has just returned to the medical unit to begin supplemental feedings through an NG tube. Which of the nurses assessments addresses this patients most significant potential complication of feeding? A) Frequent assessment of the patients abdominal girth B) Assessment for hemorrhage from the nasal insertion site C) Frequent lung auscultation D) Vigilant monitoring of the frequency and character of bowel movements

Avoid beer, especially in the evening. Elevate the head of the bed on 6- to 8-inch blocks. Elevate the upper body on pillows.

Health teaching for a patient with GERD is directed toward decreasing lower esophageal sphincter pressure and irritation. The nurse instructs the patient to do which of the following? Select all that apply.

C) It protects the stomachs lining

A nurse is providing patient education for a patient with peptic ulcer disease secondary to chronic nonsteroidal anti-inflammatory drug (NSAID) use. The patient has recently been prescribed misoprostol (Cytotec). What would the nurse be most accurate in informing the patient about the drug? A) It reduces the stomachs volume of hydrochloric acid B) It increases the speed of gastric emptying C) It protects the stomachs lining D) It increases lower esophageal sphincter pressure

asterixis

an involuntary flapping of the hands may be seen in stage 2 and cephalopathy

C) Scoliosis

A nurses assessment of a teenage girl reveals that her shoulders are not level and that she has one prominent scapula that is accentuated by bending forward. The nurse should expect to read about what health problem in the patients electronic health record? A) Lordosis B) Kyphosis C) Scoliosis D) Muscular dystrophy

D) Hospice care

A patient has been diagnosed with pancreatic cancer and has been admitted for care. Following initial treatment, the nurse should be aware that the patient is most likely to require which of the following? A) Inpatient rehabilitation B) Rehabilitation in the home setting C) Intensive physical therapy D) Hospice care

A) Colonoscopy

A patient has come to the outpatient radiology department for diagnostic testing. Which of the following diagnostic procedures will allow the care team to evaluate and remove polyps? A) Colonoscopy B) Barium enema C) ERCP D) Upper gastrointestinal fibroscopy

A) Two to 3 soft bowel movements daily

A patient has developed hepatic encephalopathy secondary to cirrhosis and is receiving care on the medical unit. The patients current medication regimen includes lactulose (Cephulac) four times daily. What desired outcome should the nurse relate to this pharmacologic intervention? A) Two to 3 soft bowel movements daily B) Significant increase in appetite and food intake C) Absence of nausea and vomiting D) Absence of blood or mucus in stool

D) A sudden release of peptides

A patient has experienced symptoms of dumping syndrome following bariatric surgery. To what physiologic phenomenon does the nurse attribute this syndrome? A) Irritation of the phrenic nerve due to diaphragmatic pressure B) Chronic malabsorption of iron and vitamins A and C C) Reflux of bile into the distal esophagus D) A sudden release of peptides

C) Placing the patient in Fowlers position

A patient has undergone surgery for oral cancer and has just been extubated in postanesthetic recovery. What nursing action best promotes comfort and facilitates spontaneous breathing for this patient? A) Placing the patient in a left lateral position B) Administering opioids as ordered C) Placing the patient in Fowlers position D) Teaching the patient to use the patient-controlled analgesia (PCA) system

B) Surgery is delayed until the acute symptoms subside.

A patient is admitted to the unit with acute cholecystitis. The physician has noted that surgery will be scheduled in 4 days. The patient asks why the surgery is being put off for a week when he has a sick gallbladder. What rationale would underlie the nurses response? A) Surgery is delayed until the patient can eat a regular diet without vomiting. B) Surgery is delayed until the acute symptoms subside. C) The patient requires aggressive nutritional support prior to surgery. D) Time is needed to determine whether a laparoscopic procedure can be used.

D) Persistently low hemoglobin and hematocrit

A patient is being assessed for a suspected deficit in intrinsic factor synthesis. What diagnostic or assessment finding is the most likely rationale for this examination of intrinsic factor production? A) Muscle wasting B) Chronic jaundice in the absence of liver disease C) The presence of fat in the patients stool D) Persistently low hemoglobin and hematocrit

A) Eating more slowly and chewing food more thoroughly

A patient is one month postoperative following restrictive bariatric surgery. The patient tells the clinic nurse that he has been having trouble swallowing for the past few days. What recommendation should the nurse make? A) Eating more slowly and chewing food more thoroughly B) Taking an OTC antacid or drinking a glass of milk prior to each meal C) Chewing gum to cause relaxation of the lower esophageal sphincter D) Drinking at least 12 ounces of liquid with each meal

B) Wash the area around the tube with soap and water daily.

A patient is postoperative day 1 following gastrostomy. The nurse is planning interventions to address the nursing diagnosis of Risk for Infection Related to Presence of Wound and Tube. What intervention is most appropriate? A) Administer antibiotics via the tube as ordered. B) Wash the area around the tube with soap and water daily. C) Cleanse the skin within 2 cm of the insertion site with hydrogen peroxide once per shift. D) Irrigate the skin surrounding the insertion site with normal saline before each use.

A) Esophageal or pyloric obstruction related to scarring

A patient was treated in the emergency department and critical care unit after ingesting bleach. What possible complication of the resulting gastritis should the nurse recognize? A) Esophageal or pyloric obstruction related to scarring B) Uncontrolled proliferation of H. pylori C) Gastric hyperacidity related to excessive gastrin secretion D) Chronic referred pain in the lower abdomen

C) Increase fluid intake to evacuate the barium

A patient who has been experiencing changes in his bowel function is scheduled for a barium enema. What instruction should the nurse provide for postprocedure recovery? A) Remain NPO for 6 hours postprocedure. B) Administer a Fleet enema to cleanse the bowel of the barium. C) Increase fluid intake to evacuate the barium. D) Avoid dairy products for 24 hours postprocedure.

A) Metoclopramide (Reglan)

A patient with GERD has undergone diagnostic testing and it has been determined that increasing the pace of gastric emptying may help alleviate symptoms. The nurse should anticipate that the patient may be prescribed what drug? A) Metoclopramide (Reglan) B) Omeprazole (Prilosec) C) Lansoprazole (Prevacid) D) Famotidine (Pepcid)

C) Respiratory status and airway clearance

A patient with cancer of the tongue has had a radical neck dissection. What nursing assessment would be a priority for this patient? A) Presence of acute pain and anxiety B) Tissue integrity and color of the operative site C) Respiratory status and airway clearance D) Self-esteem and body image

ABCD

A patient with pancreatic cancer has been scheduled for a pancreaticoduodenectomy (Whipple procedure). During health education, the patient should be informed that this procedure will involve the removal of which of the following? Select all that apply. A) Gallbladder B) Part of the stomach C) Duodenum D) Part of the common bile duct E) Part of the rectum

B) Attach a syringe filled with warm water and attempt an in-and-out motion of instilling and aspirating.

A patients NG tube has become clogged after the nurse instilled a medication that was insufficiently crushed. The nurse has attempted to aspirate with a large-bore syringe, with no success. What should the nurse do next? A) Withdraw the NG tube 3 to 5 cm and reattempt aspiration. B) Attach a syringe filled with warm water and attempt an in-and-out motion of instilling and aspirating. C) Withdraw the NG tube slightly and attempt to dislodge by flicking the tube with the fingers. D) Remove the NG tube promptly and obtain an order for reinsertion from the primary care provider.

A) Increased bilirubin

A patients abdominal ultrasound indicates cholelithiasis. When the nurse is reviewing the patients laboratory studies, what finding is most closely associated with this diagnosis? A) Increased bilirubin B) Decreased serum cholesterol C) Increased blood urea nitrogen (BUN) D) Decreased serum alkaline phosphatase level

Checking the urine for hematuria Palpating peripheral pulses in both lower extremities Testing the stool for occult blood

The nurse is caring for a patient with a pelvic fracture. What nursing assessment for a pelvic fracture should be included? (Select all that apply.)

A) Severe pancreatitis with possible peritonitis

A 37-year-old male patient presents at the emergency department (ED) complaining of nausea and vomiting and severe abdominal pain. The patients abdomen is rigid, and there is bruising to the patients flank. The patients wife states that he was on a drinking binge for the past 2 days. The ED nurse should assist in assessing the patient for what health problem? A) Severe pancreatitis with possible peritonitis B) Acute cholecystitis C) Chronic pancreatitis D) Acute appendicitis with possible perforation

D) The enzymes that your pancreas produces have damaged the pancreas itself.

A 55-year-old man has been newly diagnosed with acute pancreatitis and admitted to the acute medical unit. How should the nurse most likely explain the pathophysiology of this patients health problem? A) Toxins have accumulated and inflamed your pancreas. B) Bacteria likely migrated from your intestines and became lodged in your pancreas. C) A virus that was likely already present in your body has begun to attack your pancreatic cells. D) The enzymes that your pancreas produces have damaged the pancreas itself.

C) Use of a pressure-relieving mattress

A 91-year-old patient is slated for orthopedic surgery and the nurse is integrated gerontologic considerations into the patients plan of care. What intervention is most justified in the care of this patient? A) Administration of prophylactic antibiotics B) Total parenteral nutrition (TPN) C) Use of a pressure-relieving mattress D) Use of a Foley catheter until discharge

B) Assessment for pain

A bone biopsy has just been completed on a patient with suspected bone metastases. What assessment should the nurse prioritize in the immediate recovery period? A) Assessment for dehiscence at the biopsy site B) Assessment for pain C) Assessment for hematoma formation D) Assessment for infection

Instructing the client to remove salty and salted foods from the diet Administering prescribed spironolactone (Aldactone) Assisting with placement of a transjugular intrahepatic portosystemic shunt

A client has ascites. Which of the following interventions would the nurse prepare to assist with implementing to help the client control this condition? Select all that apply.

Streaks of blood present in the stool

A client has returned to the medical unit after a barium enema. When assessing the client's subsequent bowel patterns and stools, what finding would warrant reporting to the health care provider?

Promotion of adequate nutrition

A client has undergone rigid fixation for the correction of a mandibular fracture suffered in a fight. What area of care should the nurse prioritize when planning this client's discharge education?

Realignment of the fracture Reduction of deformity Minimization of muscle spasms

A client is placed in traction for a femur facture. The nurse would document which expected outcomes of traction? Select all that apply.

Fever, increased heart rate and decreased blood pressure

A client is receiving care in the intensive care unit for acute pancreatitis. The nurse is aware that pancreatic necrosis is a major cause of morbidity and mortality in clients with acute pancreatitis. Consequently, the nurse should assess for what signs or symptoms of this complication?

Reduced or absent bile as a result of obstruction impacts digestion.

A client is treated for gastrointestinal problems related to chronic cholecystitis. What pathophysiological process related to cholecystitis does the nurse understand is the reason behind the client's GI problems?

An effective means of communicating with the nurse

A client who had a hemiglossectomy earlier in the day is assessed postoperatively, revealing a patent airway, stable vital signs, and no bleeding or drainage from the operative site. The nurse notes the client is alert. What is the client's priority need at this time?

"Immobilization of the fracture will promote healing by maximizing contact of bone fragments." "Fractured bones require a good blood supply and adequate nutrition for healing." "Weight bearing stimulates healing of the long bones of the leg, if the fracture is stabilized."

A client who has fractured the radial head asks the nurse about factors that will promote bone healing. Which statement should the nurse include when responding to the client? Select all that apply.

"Apply a heating pad to your shoulder for 15 minutes hourly as needed."

A client who is 24 hours post op from laparoscopic cholecystectomy calls the nurse and reports pain in the right shoulder. How should the nurse respond to the client's report of symptoms?

C) Hemorrhoids

A clinic patient has described recent dark-colored stools;the nurse recognizes the need for fecal occult blood testing (FOBT). What aspect of the patients current health status would contraindicate FOBT? A) Gastroesophageal reflux disease (GERD) B) Peptic ulcers C) Hemorrhoids D) Recurrent nausea and vomiting

C) An older adult whose medication regimen includes an anticholinergic

A community health nurse serves a diverse population. What individual would likely face the highest risk for parotitis? A) A patient who is receiving intravenous antibiotic therapy in the home setting B) A patient who has a chronic venous ulcer C) An older adult whose medication regimen includes an anticholinergic D) A patient with poorly controlled diabetes who receives weekly wound care

A) Indicates acceptance of altered appearance and demonstrates positive self-image

A medical nurse who is caring for a patient being discharged home after a radical neck dissection has collaborated with the home health nurse to develop a plan of care for this patient. What is a priority psychosocial outcome for a patient who has had a radical neck dissection? A) Indicates acceptance of altered appearance and demonstrates positive self-image B) Freely expresses needs and concerns related to postoperative pain management C) Compensates effectively for alteration in ability to communicate related to dysarthria D) Demonstrates effective stress management techniques to promote muscle relaxation

B) Eat several small meals daily spaced at equal intervals.

A nurse caring for a patient who has had bariatric surgery is developing a teaching plan in anticipation of the patients discharge. Which of the following is essential to include? A) Drink a minimum of 12 ounces of fluid with each meal. B) Eat several small meals daily spaced at equal intervals. C) Choose foods that are high in simple carbohydrates. D) Sit upright when eating and for 30 minutes afterward.

A) Fowlers

A nurse in the postanesthesia care unit admits a patient following resection of a gastric tumor. Following immediate recovery, the patient should be placed in which position to facilitate patient comfort and gastric emptying? A) Fowlers B) Supine C) Left lateral D) Left Sims

D) The early symptoms of gastric cancer are usually not alarming or highly unusual.

A nurse is admitting a patient diagnosed with late-stage gastric cancer. The patients family is distraught and angry that she was not diagnosed earlier in the course of her disease. What factor contributes to the fact that gastric cancer is often detected at a later stage? A) Gastric cancer does not cause signs or symptoms until metastasis has occurred. B) Adherence to screening recommendations for gastric cancer is exceptionally low. C) Early symptoms of gastric cancer are usually attributed to constipation. D) The early symptoms of gastric cancer are usually not alarming or highly unusual.

D) Spasticity

A nurse is assessing a child who has a diagnosis of muscular dystrophy. Assessment reveals that the childs muscles have greater-than-normal tone. The nurse should document the presence of which of the following? A) Tonus B) Flaccidity C) Atony D) Spasticity

B) The patients body mass index is 34 (obese).

A nurse is assessing a patient for risk factors known to contribute to osteoarthritis. What assessment finding would the nurse interpret as a risk factor? A) The patient has a 30 pack-year smoking history. B) The patients body mass index is 34 (obese). C) The patient has primary hypertension. D) The patient is 58 years old.

B) Infection typically occurs due to ingestion of contaminated food and water.

A nurse is assessing a patient who has peptic ulcer disease. The patient requests more information about the typical causes of Helicobacter pyloriinfection. What would it be appropriate for the nurse to instruct the patient? A) Most affected patients acquired the infection during international travel. B) Infection typically occurs due to ingestion of contaminated food and water. C) Many people possess genetic factors causing a predisposition to H. pyloriinfection. D) The H. pylori microorganism is endemic in warm, moist climates.

B) Absence of feeling, capillary refill of 4 to 5 seconds, and cool

A nurse is assessing a patient who is experiencing peripheral neurovascular dysfunction. What assessment findings are most consistent with this diagnosis? A) Hot skin with a capillary refill of 1 to 2 seconds B) Absence of feeling, capillary refill of 4 to 5 seconds, and cool skin C) Pain, diaphoresis, and erythema D) Jaundiced skin, weakness, and capillary refill of 3 seconds

C) Document that the stoma appears healthy and well perfused.

A nurse is assessing a patients stoma on postoperative day 3. The nurse notes that the stoma has a shiny appearance and a bright red color. How should the nurse best respond to this assessment finding? A) Irrigate the ostomy to clear a possible obstruction. B) Contact the primary care provider to report this finding. C) Document that the stoma appears healthy and well perfused. D) Document a nursing diagnosis of Impaired Skin Integrity.

D) Signs and symptoms of septic shock

A nurse is assessing an elderly patient with gallstones. The nurse is aware that the patient may not exhibit typical symptoms, and that particular symptoms that may be exhibited in the elderly patient may include what? A) Fever and pain B) Chills and jaundice C) Nausea and vomiting D) Signs and symptoms of septic shock

C) GI diseases often produce skin changes.

A nurse is assessing the abdomen of a patient just admitted to the unit with a suspected GI disease. Inspection reveals several diverse lesions on the patients abdomen. How should the nurse best interpret this assessment finding? A) Abdominal lesions are usually due to age-related skin changes. B) Integumentary diseases often cause GI disorders. C) GI diseases often produce skin changes. D) The patient needs to be assessed for self-harm.

B) Change the dressing no more than weekly.

A nurse is aware of the high incidence of catheter-related bloodstream infections in patients receiving parenteral nutrition. What nursing action has the greatest potential to reduce catheter-related bloodstream infections? A) Use clean technique and wear a mask during dressing changes. B) Change the dressing no more than weekly. C) Apply antibiotic ointment around the site with each dressing change. D) Irrigate the insertion site with sterile water during each dressing change.

A) The patient will express satisfaction with her ability to perform ADLs.

A nurse is caring for a 78-year-old patient with a history of osteoarthritis (OA). When planning the patients care, what goal should the nurse include? A) The patient will express satisfaction with her ability to perform ADLs. B) The patient will recover from OA within 6 months. C) The patient will adhere to the prescribed plan of care. D) The patient will deny signs or symptoms of OA.

Daily weights Intake and output monitoring Calorie counts for oral nutrients

A nurse is caring for a client receiving parenteral nutrition at home. The client was discharged from the acute care facility 4 days ago. What would the nurse include in the client's plan of care? Select all that apply.

Encouraging the client to turn from side to side and to assume a prone position

A nurse is caring for a client who had a right below-the-knee amputation (BKA). The nurse recognizes the importance of implementing measures that focus on preventing flexion contracture of the hip and maintaining proper positioning. What nursing action will best achieve these goals?

To remove gas from the stomach To remove toxins from the stomach To diagnose GI motility disorders

A nurse is caring for a client who has a gastrointestinal tube in place. Which of the following are indications for gastrointestinal intubation? Select all that apply.

Acute Pain Related to Increased Peristalsis and GI Inflammation Activity Intolerance Related to Generalized Weakness Deficient Fluid Volume Related to Anorexia, Nausea, and Diarrhea

A nurse is caring for a client who has been admitted to the hospital with diverticulitis. What would be appropriate nursing diagnoses for this client? Select all that apply.

Report this finding promptly to the health care provider and remain with the client.

A nurse is caring for a client who is postoperative day 1 following neck dissection surgery. The nurse is performing an assessment of the client and notes the presence of high-pitched adventitious sounds over the client's trachea on auscultation. The client's oxygen saturation is 90% by pulse oximetry with a respiratory rate of 31 breaths per minute. What is the nurse's most appropriate action?

C) Encourage the patient to avoid scratching, and obtain an order for an antihistamine if severe itching persists.

A nurse is caring for a patient who has a leg cast. The nurse observes that the patient uses a pencil to scratch the skin under the edge of the cast. How should the nurse respond to this observation? A) Allow the patient to continue to scratch inside the cast with a pencil but encourage him to be cautious. B) Give the patient a sterile tongue depressor to use for scratching instead of the pencil. C) Encourage the patient to avoid scratching, and obtain an order for an antihistamine if severe itching persists. D) Obtain an order for a sedative, such as lorazepam (Ativan), to prevent the patient from scratching.

A) Administer an isotonic dextrose solution for 1 to 2 hours after discontinuing the PN.

A nurse is caring for a patient who has an order to discontinue the administration of parenteral nutrition. What should the nurse do to prevent the occurrence of rebound hypoglycemia in the patient? A) Administer an isotonic dextrose solution for 1 to 2 hours after discontinuing the PN. B) Administer a hypertonic dextrose solution for 1 to 2 hours after discontinuing the PN. C) Administer 3 ampules of dextrose 50% immediately prior to discontinuing the PN. D) Administer 3 ampules of dextrose 50% 1 hour after discontinuing the PN.

B) Assessment for variceal bleeding

A nurse is caring for a patient who has been admitted for the treatment of advanced cirrhosis. What assessment should the nurse prioritize in this patients plan of care? A) Measurement of abdominal girth and body weight B) Assessment for variceal bleeding C) Assessment for signs and symptoms of jaundice D) Monitoring of results of liver function testing

ABD

A nurse is caring for a patient who has been admitted to the hospital with diverticulitis. Which of the following would be appropriate nursing diagnoses for this patient? Select all that apply. A) Acute Pain Related to Increased Peristalsis and GI Inflammation B) Activity Intolerance Related to Generalized Weakness C) Bowel Incontinence Related to Increased Intestinal Peristalsis D) Deficient Fluid Volume Related to Anorexia, Nausea, and Diarrhea E) Impaired Urinary Elimination Related to GI Pressure on the Bladder

C) Youll be encouraged to drink water after the administration of the radioisotope injection.

A nurse is caring for a patient who has been scheduled for a bone scan. What should the nurse teach the patient about this diagnostic test? A) The test is brief and requires that you drink a calcium solution 2 hours before the test. B) You will not be allowed fluid for 2 hours before and 3 hours after the test. C) Youll be encouraged to drink water after the administration of the radioisotope injection. D) This is a common test that can be safely performed on anyone.

B) The use of moderate sedation

A nurse is caring for a patient who has been scheduled for endoscopic retrograde cholangiopancreatography (ERCP) the following day. When providing anticipatory guidance for this patient, the nurse should describe what aspect of this diagnostic procedure? A) The need to protect the incision postprocedure B) The use of moderate sedation C) The need to infuse 50% dextrose during the procedure D) The use of general anesthesia

B) Avoiding chewing food for the specified number of weeks after surgery

A nurse is caring for a patient who has just had a rigid fixation of a mandibular fracture. When planning the discharge teaching for this patient, what would the nurse be sure to include? A) Increasing calcium intake to promote bone healing B) Avoiding chewing food for the specified number of weeks after surgery C) Techniques for managing parenteral nutrition in the home setting D) Techniques for managing a gastrostomy

D) Report this finding promptly to the physician and remain with the patient.

A nurse is caring for a patient who is postoperative day 1 following neck dissection surgery. The nurse is performing an assessment of the patient and notes the presence of high-pitched adventitious sounds over the patients trachea on auscultation. The patients oxygen saturation is 90% by pulse oximetry with a respiratory rate of 31 breaths per minute. What is the nurses most appropriate action? A) Encourage the patient to perform deep breathing and coughing exercises hourly. B) Reposition the patient into a prone or semi-Fowlers position and apply supplementary oxygen by nasal cannula. C) Activate the emergency response system. D) Report this finding promptly to the physician and remain with the patient.

A) Encourage the family to bring in the patients favored foods.

A nurse is caring for a patient who is postoperative from a neck dissection. What would be the most appropriate nursing action to enhance the patients appetite? A) Encourage the family to bring in the patients favored foods. B) Limit visitors at mealtimes so that the patient is not distracted. C) Avoid offering food unless the patient initiates. D) Provide thorough oral care immediately after the patient eats.

A) Inflammatory bowel disease

A nurse is caring for a patient who is scheduled for a colonoscopy and whose bowel preparation will include polyethylene glycol electrolyte lavage prior to the procedure. The presence of what health problem would contraindicate the use of this form of bowel preparation? A) Inflammatory bowel disease B) Intestinal polyps C) Diverticulitis D) Colon cancer

A) A dull, deep ache that is boring in nature

A nurse is caring for a patient whose cancer metastasis has resulted in bone pain. Which of the following are typical characteristics of bone pain? A) A dull, deep ache that is boring in nature B) Soreness or aching that may include cramping C) Sharp, piercing pain that is relieved by immobilization D) Spastic or sharp pain that radiates

D) Report possible signs of aspiration pneumonia to the primary care provider.

A nurse is caring for a patient with a nasogastric tube for feeding. During shift assessment, the nurse auscultates a new onset of bilateral lung crackles and notes a respiratory rate of 30 breaths per minute. The patients oxygen saturation is 89% by pulse oximetry. After ensuring the patients immediate safety, what is the nurses most appropriate action? A) Perform chest physiotherapy. B) Reduce the height of the patients bed and remove the NG tube. C) Liaise with the dietitian to obtain a feeding solution with lower osmolarity. D) Report possible signs of aspiration pneumonia to the primary care provider.

B) Avoid taking the drug on a long-term basis.

A nurse is caring for a patient with constipation whose primary care provider has recommended senna (Senokot) for the management of this condition. The nurse should provide which of the following education points? A) Limit your fluid intake temporarily so you dont get diarrhea. B) Avoid taking the drug on a long-term basis. C) Make sure to take a multivitamin with each dose. D) Take this on an empty stomach to ensure maximum effect.

A) Stomach emptying takes place more slowly.

A nurse is caring for an 83-year-old patient who is being assessed for recurrent and intractable nausea. What age-related change to the GI system may be a contributor to the patients health complaint? A) Stomach emptying takes place more slowly. B) The villi and epithelium of the small intestine become thinner. C) The esophageal sphincter becomes incompetent. D) Saliva production decreases.

D) Potassium level

A nurse is caring for an older adult who has been experiencing severeClostridium difficile-related diarrhea. When reviewing the patients most recent laboratory tests, the nurse should prioritize which of the following? A) White blood cell level B) Creatinine level C) Hemoglobin level D) Potassium level

Ans: B, C, D, E

A nurse is creating a care plan for a patient who is receiving parenteral nutrition. The patients care plan should include nursing actions relevant to what potential complications? Select all that apply. A) Dumping syndrome B) Clotted or displaced catheter C) Pneumothorax D) Hyperglycemia E) Line sepsis

B) Initiating the infusion slowly and monitoring the patients fluid and glucose tolerance

A nurse is initiating parenteral nutrition (PN) to a postoperative patient who has developed complications. The nurse should initiate therapy by performing which of the following actions? A) Starting with a rapid infusion rate to meet the patients nutritional needs as quickly as possible B) Initiating the infusion slowly and monitoring the patients fluid and glucose tolerance C) Changing the rate of administration every 2 hours based on serum electrolyte values D) Increasing the rate of infusion at mealtimes to mimic the circadian rhythm of the body

A nurse is participating in the emergency care of a client who has just developed variceal bleeding. What intervention should the nurse anticipate?

A nurse is participating in the emergency care of a client who has just developed variceal bleeding. What intervention should the nurse anticipate?

D) Decreased mucus secretion

A nurse is performing an abdominal assessment of an older adult patient. When collecting and analyzing data, the nurse should be cognizant of what age-related change in gastrointestinal structure and function? A) Increased gastric motility B) Decreased gastric pH C) Increased gag reflex D) Decreased mucus secretion

C) Place hand under right lower rib cage and press down lightly with the other hand.

A nurse is performing an admission assessment of a patient with a diagnosis of cirrhosis. What technique should the nurse use to palpate the patients liver? A) Place hand under the right lower abdominal quadrant and press down lightly with the other hand. B) Place the left hand over the abdomen and behind the left side at the 11th rib. C) Place hand under right lower rib cage and press down lightly with the other hand. D) Hold hand 90 degrees to right side of the abdomen and push down firmly.

C) Joint stiffness, especially in the morning

A nurse is performing the health history and physical assessment of a patient who has a diagnosis of rheumatoid arthritis (RA). What assessment finding is most consistent with the clinical presentation of RA? A) Cool joints with decreased range of motion B) Signs of systemic infection C) Joint stiffness, especially in the morning D) Visible atrophy of the knee and shoulder joints

C) The familys ability to provide emotional support

A nurse is planning discharge teaching for a 21-year-old patient with a new diagnosis of ulcerative colitis. When planning family assessment, the nurse should recognize that which of the following factors will likely have the greatest impact on the patients coping after discharge? A) The familys ability to take care of the patients special diet needs B) The familys ability to monitor the patients changing health status C) The familys ability to provide emotional support D) The familys ability to manage the patients medication regimen

B) Visual changes

A nurse is planning patient education for a patient being discharged home with a diagnosis of rheumatoid arthritis. The patient has been prescribed antimalarials for treatment, so the nurse knows to teach the patient to self-monitor for what adverse effect? A) Tinnitus B) Visual changes C) Stomatitis D) Hirsutism

Regular bone density testing A high-calcium diet Use of falls prevention precautions Weight-bearing exercise

A nurse is planning the care of an older adult client who will soon be discharged home after treatment for a fractured hip. In an effort to prevent future fractures, the nurse should encourage what actions? Select all that apply.

D) Contact the pharmacy to obtain a new bag of PN.

A nurse is preparing to administer a patients scheduled parenteral nutrition (PN). Upon inspecting the bag, the nurse notices that the presence of small amounts of white precipitate are present in the bag. What is the nurses best action? A) Recognize this as an expected finding. B) Place the bag in a warm environment for 30 minutes. C) Shake the bag vigorously for 10 to 20 seconds. D) Contact the pharmacy to obtain a new bag of PN.

ABC

A nurse is preparing to discharge a patient home on parenteral nutrition. What should an effective home care teaching program address? Select all that apply. A) Preparing the patient to troubleshoot for problems B) Teaching the patient and family strict aseptic technique C) Teaching the patient and family how to set up the infusion D) Teaching the patient to flush the line with sterile water E) Teaching the patient when it is safe to leave the access site open to air

A) Pepsin

A nurse is promoting increased protein intake to enhance a patients wound healing. The nurse knows that enzymes are essential in the digestion of nutrients such as protein. What is the enzyme that initiates the digestion of protein? A) Pepsin B) Intrinsic factor C) Lipase D) Amylase

C) Youll need to have enemas the day before the test.

A nurse is providing preprocedure education for a patient who will undergo a lower GI tract study the following week. What should the nurse teach the patient about bowel preparation? A) Youll need to fast for at least 18 hours prior to your test. B) Starting today, take over-the-counter stool softeners twice daily. C) Youll need to have enemas the day before the test. D) For 24 hours before the test, insert a glycerin suppository every 4 hours.

A) Evaluating the effects of the musculoskeletal disorder on the patients function

A nurse is taking a health history on a patient with musculoskeletal dysfunction. What is the primary focus of this phase of the nurses assessment? A) Evaluating the effects of the musculoskeletal disorder on the patients function B) Evaluating the patients adherence to the existing treatment regimen C) Evaluating the presence of genetic risk factors for further musculoskeletal disorders D) Evaluating the patients active and passive range of motion

Maintain an open dialogue with the client and facilitate a referral to the wound-ostomy-continence (WOC) nurse.

A nurse is talking with a client who is scheduled to have a hemicolectomy with the creation of a colostomy. The client admits to being anxious, and has many questions concerning the surgery, the care of a stoma, and necessary lifestyle changes. What nursing action is most appropriate?

C) A 39-year-old man with chronic alcoholism

A nurse who provides care in a walk-in clinic assesses a wide range of individuals. The nurse should identify which of the following patients as having the highest risk for chronic pancreatitis? A) A 45-year-old obese woman with a high-fat diet B) An 18-year-old man who is a weekend binge drinker C) A 39-year-old man with chronic alcoholism D) A 51-year-old woman who smokes one-and-a-half packs of cigarettes per day

A) A 65-year-old man with alcoholism who smokes

A nurse who provides care in an ambulatory clinic integrates basic cancer screening into admission assessments. What patient most likely faces the highest immediate risk of oral cancer? A) A 65-year-old man with alcoholism who smokes B) A 45-year-old woman who has type 1 diabetes and who wears dentures C) A 32-year-old man who is obese and uses smokeless tobacco D) A 57-year-old man with GERD and dental caries

48-year-old female with BMI 36 k/m2 and uncontrolled type 2 diabetes. 34-year-old male with BMI 30 k/m2 and metabolic syndrome with hypertension.

A nurse working in a bariatric clinic assesses various clients with obesity. Which clients will the nurse recognize as meeting the selction criteria for bariatric surgery? Select all that apply.

B) Prevent aspiration

A nursing educator is reviewing the care of patients with feeding tubes and endotracheal tubes (ET). The educator has emphasized the need to check for tube placement in the stomach as well as residual volume. What is the main purpose of this nursing action? A) Prevent gastric ulcers B) Prevent aspiration C) Prevent abdominal distention D) Prevent diarrhea

B) A pregnant woman at 28 weeks gestation

A nursing instructor is discussing hemorrhoids with the nursing class. Which patients would the nursing instructor identify as most likely to develop hemorrhoids? A) A 45-year-old teacher who stands for 6 hours per day B) A pregnant woman at 28 weeks gestation C) A 37-year-old construction worker who does heavy lifting D) A 60-year-old professional who is under stress

B) Contact the primary care provider promptly and report these signs of perforation.

A patient admitted with acute diverticulitis has experienced a sudden increase in temperature and complains of a sudden onset of exquisite abdominal tenderness. The nurses rapid assessment reveals that the patients abdomen is uncharacteristically rigid on palpation. What is the nurses best response? A) Administer a Fleet enema as ordered and remain with the patient. B) Contact the primary care provider promptly and report these signs of perforation. C) Position the patient supine and insert an NG tube. D) Page the primary care provider and report that the patient may be obstructed.

B) Insertion of an NG tube for decompression

A patient has been admitted to the hospital after diagnostic imaging revealed the presence of a gastric outlet obstruction (GOO). What is the nurses priority intervention? A) Administration of antiemetics B) Insertion of an NG tube for decompression C) Infusion of hypotonic IV solution D) Administration of proton pump inhibitors as ordered

D) Metastases are common and respond poorly to treatment.

A patient has been diagnosed with a malignancy of the oral cavity and is undergoing oncologic treatment. The oncologic nurse is aware that the prognosis for recovery from head and neck cancers is often poor because of what characteristic of these malignancies? A) Radiation therapy often results in secondary brain tumors. B) Surgical complications are exceedingly common. C) Diagnosis rarely occurs until the cancer is endstage. D) Metastases are common and respond poorly to treatment.

D) Maintaining fluid and electrolyte balance

A patient has been diagnosed with a small bowel obstruction and has been admitted to the medical unit. The nurses care should prioritize which of the following outcomes? A) Preventing infection B) Maintaining skin and tissue integrity C) Preventing nausea and vomiting D) Maintaining fluid and electrolyte balance

B) Abdominal pain and hepatomegaly

A patient has been diagnosed with advanced stage breast cancer and will soon begin aggressive treatment. What assessment findings would most strongly suggest that the patient may have developed liver metastases? A) Persistent fever and cognitive changes B) Abdominal pain and hepatomegaly C) Peripheral edema unresponsive to diuresis D) Spontaneous bleeding and jaundice

B) Regurgitation of undigested food

A patient has been diagnosed with an esophageal diverticulum after undergoing diagnostic imaging. When taking the health history, the nurse should expect the patient to describe what sign or symptom? A) Burning pain on swallowing B) Regurgitation of undigested food C) Symptoms mimicking a heart attack D) Chronic parotid abscesses

ABD

A patient has been discharged home on parenteral nutrition (PN). Much of the nurses discharge education focused on coping. What must a patient on PN likely learn to cope with? Select all that apply. A) Changes in lifestyle B) Loss of eating as a social behavior C) Chronic bowel incontinence from GI changes D) Sleep disturbances related to frequent urination during nighttime infusions E) Stress of choosing the correct PN formulation

A) Injection of a contrast agent into the knee joint prior to ROM exercises

A patient has been experiencing an unexplained decline in knee function and has consequently been scheduled for arthrography. The nurse should teach the patient about what process? A) Injection of a contrast agent into the knee joint prior to ROM exercises B) Aspiration of synovial fluid for serologic testing C) Injection of corticosteroids into the patients knee joint to facilitate ROM D) Replacement of the patients synovial fluid with a synthetic substitute

D) Frequent screening for osteoporosis

A patient has been experiencing disconcerting GI symptoms that have been worsening in severity. Following medical assessment, the patient has been diagnosed with lactose intolerance. The nurse should recognize an increased need for what form of health promotion? A) Annual screening colonoscopies B) Adherence to recommended immunization schedules C) Regular blood pressure monitoring D) Frequent screening for osteoporosis

D) Psyllium hydrophilic mucilloid (Metamucil)

A patient has been experiencing occasional episodes of constipation and has been unable to achieve consistent relief by increasing physical activity and improving his diet. What pharmacologic intervention should the nurse recommend to the patient for ongoing use? A) Mineral oil enemas B) Bisacodyl (Dulcolax) C) Senna (Senokot) D) Psyllium hydrophilic mucilloid (Metamucil)

C) Arthrocentesis

A patient has been experiencing progressive increases in knee pain and diagnostic imaging reveals a worsening effusion in the synovial capsule. The nurse should anticipate which of the following? A) Arthrography B) Knee biopsy C) Arthrocentesis D) Electromyography

B) Risk For Impaired Skin Integrity Related to Peptic Ulcers

A patient has been scheduled for a urea breath test in one months time. What nursing diagnosis most likely prompted this diagnostic test? A) Impaired Dentition Related to Gingivitis B) Risk For Impaired Skin Integrity Related to Peptic Ulcers C) Imbalanced Nutrition: Less Than Body Requirements Related to Enzyme Deficiency D) Diarrhea Related to Clostridium Difficile Infection

B) A quantitative fecal immunochemical test

A patient has come to the clinic complaining of blood in his stool. A FOBT test is performed but is negative. Based on the patients history, the physician suggests a colonoscopy, but the patient refuses, citing a strong aversion to the invasive nature of the test. What other test might the physician order to check for blood in the stool? A) A laparoscopic intestinal mucosa biopsy B) A quantitative fecal immunochemical test C) Computed tomography (CT) D) Magnetic resonance imagery (MRI)

D) Contact the primary care provider immediately.

A patient has had a cast placed for the treatment of a humeral fracture. The nurses most recent assessment shows signs and symptoms of compartment syndrome. What is the nurses most appropriate action? A) Arrange for a STAT assessment of the patients serum calcium levels. B) Perform active range of motion exercises. C) Assess the patients joint function symmetrically. D) Contact the primary care provider immediately.

D) Providing the patient with physical and emotional support

A patient has just been diagnosed with acute gastritis after presenting in distress to the emergency department with abdominal symptoms. What would be the nursing care most needed by the patient at this time? A) Teaching the patient about necessary nutritional modification B) Helping the patient weigh treatment options C) Teaching the patient about the etiology of gastritis D) Providing the patient with physical and emotional support

A) Wrap the joint in a compression dressing.

A patient has just had an arthroscopy performed to assess a knee injury. What nursing intervention should the nurse implement following this procedure? A) Wrap the joint in a compression dressing. B) Perform passive range of motion exercises. C) Maintain the knee in flexion for up to 30 minutes. D) Apply heat to the knee.

A) Ensure that none of the patients visitors has an infection.

A patient has received treatment for oral cancer. The combination of medications and radiotherapy has resulted in leukopenia. Which of the following is an appropriate response to this change in health status? A) Ensure that none of the patients visitors has an infection. B) Arrange for a diet that is high in protein and low in fat. C) Administer colony stimulating factors (CSFs) as ordered. D) Prepare to administer chemotherapeutics as ordered.

B) Document this as an expected assessment finding.

A patient has recently been admitted to the orthopedic unit following total hip arthroplasty. The patient has a closed suction device in place and the nurse has determined that there were 320 mL of output in the first 24 hours. How should the nurse best respond to this assessment finding? A) Inform the primary care provider promptly. B) Document this as an expected assessment finding. C) Limit the patients fluid intake to 2 liters for the next 24 hours. D) Administer a loop diuretic as ordered.

A) NSAIDs

A patient has sought care because of recent dark-colored stools. As a result, a fecal occult blood test has been ordered. The nurse should instruct the patient to avoid which of the following prior to collecting a stool sample? A) NSAIDs B) Acetaminophen C) OTC vitamin D supplements D) Fiber supplements

C) Signs and symptoms of intra-abdominal complications

A patient has undergone a laparoscopic cholecystectomy and is being prepared for discharge home. When providing health education, the nurse should prioritize which of the following topics? A) Management of fluid balance in the home setting B) The need for blood glucose monitoring for the next week C) Signs and symptoms of intra-abdominal complications D) Appropriate use of prescribed pancreatic enzymes

C) Promotion of adequate nutrition

A patient has undergone rigid fixation for the correction of a mandibular fracture suffered in a fight. What area of care should the nurse prioritize when planning this patients discharge education? A) Resumption of activities of daily living B) Pain control C) Promotion of adequate nutrition D) Strategies for promoting communication

C) Bucks extension traction

A patient is admitted to the unit in traction for a fractured proximal femur and requires traction prior to surgery. What is the most appropriate type of traction to apply to a fractured proximal femur? A) Russells traction B) Dunlops traction C) Bucks extension traction D) Cervical head halter

D) Make appropriate referrals to services that provide psychosocial support.

A patient is recovering in the hospital following gastrectomy. The nurse notes that the patient has become increasingly difficult to engage and has had several angry outbursts at various staff members in recent days. The nurses attempts at therapeutic dialogue have been rebuffed. What is the nurses most appropriate action? A) Ask the patients primary care provider to liaise between the nurse and the patient. B) Delegate care of the patient to a colleague. C) Limit contact with the patient in order to provide privacy. D) Make appropriate referrals to services that provide psychosocial support.

B) That the patient emptied the bladder

A patient is scheduled for a bone scan to rule out osteosarcoma of the pelvic bones. What would be most important for the nurse to assess before the patients scan? A) That the patient completed the bowel cleansing regimen B) That the patient emptied the bladder C) That the patient is not allergic to penicillins D) That the patient has fasted for at least 8 hours

B) Altered serum calcium levels

A patient is undergoing diagnostic testing for suspected Pagets disease. What assessment finding is most consistent with this diagnosis? A) Altered serum magnesium levels B) Altered serum calcium levels C) Altered serum potassium levels D) Altered serum sodium levels

A) OA is a considered a noninflammatory joint disease. RA is characterized by inflamed, swollen joints.

A patient is undergoing diagnostic testing to determine the etiology of recent joint pain. The patient asks the nurse about the difference between osteoarthritis (OA) and rheumatoid arthritis (RA). What is the best response by the nurse? A) OA is a considered a noninflammatory joint disease. RA is characterized by inflamed, swollen joints. B) OA and RA are very similar. OA affects the smaller joints such as the fingers, and RA affects the larger, weight-bearing joints like the knees. C) OA originates with an infection. RA is a result of your bodys cells attacking one another. D) OA is associated with impaired immune function; RA is a consequence of physical damage.

C) Dilute the concentration of the feeding solution.

A patient receiving tube feedings is experiencing diarrhea. The nurse and the physician suspect that the patient is experiencing dumping syndrome. What intervention is most appropriate? A) Stop the tube feed and aspirate stomach contents. B) Increase the hourly feed rate so it finishes earlier. C) Dilute the concentration of the feeding solution. D) Administer fluid replacement by IV.

A) Tachycardia, hypotension, and tachypnea

A patient who experienced an upper GI bleed due to gastritis has had the bleeding controlled and the patients condition is now stable. For the next several hours, the nurse caring for this patient should assess for what signs and symptoms of recurrence? A) Tachycardia, hypotension, and tachypnea B) Tarry, foul-smelling stools C) Diaphoresis and sudden onset of abdominal pain D) Sudden thirst, unrelieved by oral fluid administration

B) An effective means of communicating with the nurse

A patient who had a hemiglossectomy earlier in the day is assessed postoperatively, revealing a patent airway, stable vital signs, and no bleeding or drainage from the operative site. The nurse notes the patient is alert. What is the patients priority need at this time? A) Emotional support from visitors and staff B) An effective means of communicating with the nurse C) Referral to a speech therapist D) Dietary teaching focused on consistency of food and frequency of feedings

C) Positioning the patient to prevent gastric reflux

A patient who underwent surgery for esophageal cancer is admitted to the critical care unit following postanesthetic recovery. Which of the following should be included in the patients immediate postoperative plan of care? A) Teaching the patient to self-suction B) Performing chest physiotherapy to promote oxygenation C) Positioning the patient to prevent gastric reflux D) Providing a regular diet as tolerated

B) Frequent abdominal auscultation

A patient with a diagnosis of colon cancer is 2 days postoperative following bowel resection and anastomosis. The nurse has planned the patients care in the knowledge of potential complications. What assessment should the nurse prioritize? A) Close monitoring of temperature B) Frequent abdominal auscultation C) Assessment of hemoglobin, hematocrit, and red blood cell levels D) Palpation of peripheral pulses and leg girth

B) Keep patient NPO until the patients gag reflex returns.

A patient with a diagnosis of esophageal varices has undergone endoscopy to gauge the progression of this complication of liver disease. Following the completion of this diagnostic test, what nursing intervention should the nurse perform? A) Keep patient NPO until the results of test are known. B) Keep patient NPO until the patients gag reflex returns. C) Administer analgesia until post-procedure tenderness is relieved. D) Give the patient a cold beverage to promote swallowing ability.

A) This medication will reduce the amount of acid secreted in your stomach.

A patient with a diagnosis of peptic ulcer disease has just been prescribed omeprazole (Prilosec). How should the nurse best describe this medications therapeutic action? A) This medication will reduce the amount of acid secreted in your stomach. B) This medication will make the lining of your stomach more resistant to damage. C) This medication will specifically address the pain that accompanies peptic ulcer disease. D) This medication will help your stomach lining to repair itself.

B) The patient has a rigid, boardlike abdomen that is tender.

A patient with a history of peptic ulcer disease has presented to the emergency department (ED) in distress. What assessment finding would lead the ED nurse to suspect that the patient has a perforated ulcer? A) The patient has abdominal bloating that developed rapidly. B) The patient has a rigid, boardlike abdomen that is tender. C) The patient is experiencing intense lower right quadrant pain. D) The patient is experiencing dizziness and confusion with no apparent hemodynamic changes.

D) Avoid drinking alcohol while taking the drug.

A patient with a peptic ulcer disease has had metronidazole (Flagyl) added to his current medication regimen. What health education related to this medication should the nurse provide? A) Take the medication on an empty stomach. B) Take up to one extra dose per day if stomach pain persists. C) Take at bedtime to mitigate the effects of drowsiness. D) Avoid drinking alcohol while taking the drug.

B) Keep your right leg elevated above heart level

A patient with a right tibial fracture is being discharged home after having a cast applied. What instruction should the nurse provide in relationship to the patients cast care? A) Cover the cast with a blanket until the cast dries. B) Keep your right leg elevated above heart level. C) Use a clean object to scratch itches inside the cast. D) A foul smell from the cast is normal after the first few days.

B) Assess the surgical site and the affected extremity.

A patient with a total hip replacement is progressing well and expects to be discharged tomorrow. On returning to bed after ambulating, he complains of a new onset of pain at the surgical site. What is the nurses best action? A) Administer pain medication as ordered. B) Assess the surgical site and the affected extremity. C) Reassure the patient that pain is a direct result of increased activity. D) Assess the patient for signs and symptoms of systemic infection.

A) You seem like youre feeling angry. Is that something that we could talk about?

A patient with an exacerbation of systemic lupus erythematosus (SLE) has been hospitalized on the medical unit. The nurse observes that the patient expresses angerand irritation when her call bell isnt answered immediately. What would be the most appropriate response? A) You seem like youre feeling angry. Is that something that we could talk about? B) Try to remember that stress can make your symptoms worse. C) Would you like to talk about the problem with the nursing supervisor? D) I can see youre angry. Ill come back when youve calmed down.

B) Pain relief occurs by 6 months in most patients who undergo this procedure, but some people experience a recurrence of their pain.

A patient with chronic pancreatitis had a pancreaticojejunostomy created 3 months ago for relief of pain and to restore drainage of pancreatic secretions. The patient has come to the office for a routine postsurgical appointment. The patient is frustrated that the pain has not decreased. What is the most appropriate initial response by the nurse? A) The majority of patients who have a pancreaticojejunostomy have their normal digestion restored but do not achieve pain relief. B) Pain relief occurs by 6 months in most patients who undergo this procedure, but some people experience a recurrence of their pain. C) Your physician will likely want to discuss the removal of your gallbladder to achieve pain relief. D) You are probably not appropriately taking the medications for your pancreatitis and pain, so we will need to discuss your medication regimen in detail.

A) The patient will require an upper endoscopy every 6 months to detect malignant changes.

A patient with gastroesophageal reflux disease (GERD) has a diagnosis of Barretts esophagus with minor cell changes. Which of the following principles should be integrated into the patients subsequent care? A) The patient will require an upper endoscopy every 6 months to detect malignant changes. B) Liver enzymes must be checked regularly, as H2 receptor antagonists may cause hepatic damage. C) Small amounts of blood are likely to be present in the stools and are not cause for concern. D) Antacids may be discontinued when symptoms of heartburn subside.

C) Reduction in sodium intake

A patient with liver disease has developed jaundice; the nurse is collaborating with the patient to develop a nutritional plan. The nurse should prioritize which of the following in the patients plan? A) Increased potassium intake B) Fluid restriction to 2 L per day C) Reduction in sodium intake D) High-protein, low-fat diet

B) Entry of large amounts of water into the small intestine because of osmotic pressure

A patients enteral feedings have been determined to be too concentrated based on the patients development of dumping syndrome. What physiologic phenomenon caused this patients complication of enteral feeding? A) Increased gastric secretion of HCl and gastrin because of high osmolality of feeds B) Entry of large amounts of water into the small intestine because of osmotic pressure C) Mucosal irritation of the stomach and small intestine by the high concentration of the feed D) Acidbase imbalance resulting from the high volume of solutes in the feed

A) Chemical phlebitis

A patients health decline necessitates the use of total parenteral nutrition. The patient has questioned the need for insertion of a central venous catheter, expressing a preference for a normal IV. The nurse should know that peripheral administration of high-concentration PN formulas is contraindicated because of the risk for what complication? A) Chemical phlebitis B) Hyperglycemia C) Dumping syndrome D) Line sepsis

C) An absence of blood in stool

A patients health history is suggestive of inflammatory bowel disease. Which of the following would suggest Crohns disease, rather that ulcerative colitis, as the cause of the patients signs and symptoms? A) A pattern of distinct exacerbations and remissions B) Severe diarrhea C) An absence of blood in stool D) Involvement of the rectal mucosa

A) Confirm placement of the tube prior to each medication administration.

A patients new onset of dysphagia has required insertion of an NG tube for feeding; the nurse has modified the patients care plan accordingly. What intervention should the nurse include in the patients plan of care? A) Confirm placement of the tube prior to each medication administration. B) Have the patient sip cool water to stimulate saliva production. C) Keep the patient in a low Fowlers position when at rest. D) Connect the tube to continuous wall suction when not in use.

B) Nontunneled central catheter

A patients physician has determined that for the next 3 to 4 weeks the patient will require parenteral nutrition (PN). The nurse should anticipate the placement of what type of venous access device? A) Peripheral catheter B) Nontunneled central catheter C) Implantable port D) Tunneled central catheter

B) The patient can resume a normal routine immediately.

A patients sigmoidoscopy has been successfully completed and the patient is preparing to return home. Which of the following teaching points should the nurse include in the patients discharge education? A) The patient should drink at least 2 liters of fluid in the next 12 hours. B) The patient can resume a normal routine immediately. C) The patient should expect fecal urgency for several hours. D) The patient can expect some scant rectal bleeding.

D) Assessing the patency of the ulnar artery

A radial graft is planned in the treatment of a patients oropharyngeal cancer. In order to ensure that the surgery will be successful, the care team must perform what assessment prior to surgery? A) Assessing function of cranial nerves V, VI, and IX B) Assessing for a history of GERD C) Assessing for signs or symptoms of atherosclerosis D) Assessing the patency of the ulnar artery

B) Lower esophageal sphincter

A staff educator is reviewing the causes of gastroesophageal reflux disease (GERD) with new staff nurses. What area of the GI tract should the educator identify as the cause of reduced pressure associated with GERD? A) Pyloric sphincter B) Lower esophageal sphincter C) Hypopharyngeal sphincter D) Upper esophageal sphincter

A) Palpate the patients parotid glands to detect swelling and tenderness.

An elderly patient comes into the emergency department complaining of an earache. The patient and has an oral temperature of 100.2F and otoscopic assessment of the ear reveals a pearly gray tympanic membrane with no evidence of discharge or inflammation. Which action should the triage nurse take next? A) Palpate the patients parotid glands to detect swelling and tenderness. B) Assess the temporomandibular joint for evidence of a malocclusion. C) Test the integrity of cranial nerve XII by asking the patient to protrude the tongue. D) Inspect the patients gums for bleeding and hyperpigmentation.

A) Bone densitometry

An older adult patient has symptoms of osteoporosis and is being assessed during her annual physical examination. The assessment shows that the patient will require further testing related to a possible exacerbation of her osteoporosis. The nurse should anticipate what diagnostic test? A) Bone densitometry B) Hip bone radiography C) Computed tomography (CT) D) Magnetic resonance imaging (MRI)

"Often the area of pain is referred from another area."

The nurse asks a client to point to where pain is felt. The client asks why this is important. What is the nurse's best response?

Monitoring the feeding closely.

The nurse cares for a client who receivies continuous enteral tube feedings and who is at low risk for aspiration. The nurse assesses the gastric residual volume to be 350 mL. The nurse determines which action is correct?

Splenic vein Inferior mesenteric vein Gastric vein

The nurse educator is reviewing the blood supply of the GI tract with a group of medical nurses. The nurse is explaining the fact that the veins that return blood from the digestive organs and the spleen form the portal venous system. What large veins will the nurse list when describing this system? Select all that apply.

ABC

The nurse educator is reviewing the blood supply of the GI tract with a group of medical nurses. The nurse is explaining the fact that the veins that return blood from the digestive organs and the spleen form the portal venous system. What large veins will the nurse list when describing this system? Select all that apply. A) Splenic vein B) Inferior mesenteric vein C) Gastric vein D) Inferior vena cava E) Saphenous vein

A) Checking the patients capillary blood glucose levels regularly

The nurse is administering total parenteral nutrition (TPN) to a client who underwent surgery for gastric cancer. Which of the nurses assessments most directly addresses a major complication of TPN? A) Checking the patients capillary blood glucose levels regularly B) Having the patient frequently rate his or her hunger on a 10-point scale C) Measuring the patients heart rhythm at least every 6 hours D) Monitoring the patients level of consciousness each shift

A) Within 30 minutes, then every 1 to 2 hours

The nurse is helping to set up Bucks traction on an orthopedic patient. How often should the nurse assess circulation to the affected leg? A) Within 30 minutes, then every 1 to 2 hours B) Within 30 minutes, then every 4 hours C) Within 30 minutes, then every 8 hours D) Within 30 minutes, then every shift

Keep the vent lumen above the patient's waist to prevent gastric content reflux.

The nurse is managing a gastric (Salem) sump tube for a patient who has an intestinal obstruction and will be going to surgery. What interventions should the nurse perform to make sure the tube is functioning properly?

Pale, cyanotic, or mottled color Cool temperature of the extremity More than 3-second capillary refill

The nurse is performing an assessment for a patient who may have peripheral neurovascular dysfunction. What signs does the patient present with that indicate circulation is impaired? (Select all that apply.)

The absorption into the bloodstream of nutrient molecules produced by digestion

The nurse is providing health education to a client with a gastrointestinal disorder. What should the nurse describe as a major function of the GI tract?

C) Lying on the left side with legs drawn toward the chest

The nurse is providing health education to a patient scheduled for a colonoscopy. The nurse should explain that she will be placed in what position during this diagnostic test? A) In a knee-chest position (lithotomy position) B) Lying prone with legs drawn toward the chest C) Lying on the left side with legs drawn toward the chest D) In a prone position with two pillows elevating the buttocks

A) Avoid applying suction on or near the suture line.

The nurse notes that a patient who has undergone skin, tissue, and muscle grafting following a modified radical neck dissection requires suctioning. What is the most important consideration for the nurse when suctioning this patient? A) Avoid applying suction on or near the suture line. B) Position patient on the non operative side with the head of the bed down. C) Assess the patients ability to perform self-suctioning. D) Evaluate the patients ability to swallow saliva and clear fluids.

B) Presence of a painless sore with raised edges

The nurses comprehensive assessment of a patient includes inspection for signs of oral cancer. What assessment finding is most characteristic of oral cancer in its early stages? A) Dull pain radiating to the ears and teeth B) Presence of a painless sore with raised edges C) Areas of tenderness that make chewing difficult D) Diffuse inflammation of the buccal mucosa

C) Instruct the patient to walk away from the nurse for a short distance and then toward the nurse.

The nurses comprehensive assessment of an older adult involves the assessment of the patients gait. How should the nurse best perform this assessment? A) Instruct the patient to walk heel-to-toe for 15 to 20 steps. B) Instruct the patient to walk in a straight line while not looking at the floor. C) Instruct the patient to walk away from the nurse for a short distance and then toward the nurse. D) Instruct the patient to balance on one foot for as long as possible and then walk in a circle around the room.

A) Increased warmth of the calf

The nursing care plan for a patient in traction specifies regular assessments for venous thromboembolism (VTE). When assessing a patients lower limbs, what sign or symptom is suggestive of deep vein thrombosis (DVT)? A) Increased warmth of the calf B) Decreased circumference of the calf C) Loss of sensation to the calf D) Pale-appearing calf

B) Balanced traction allows for greater patient movement and independence than other forms of traction.

The orthopedic surgeon has prescribed balanced skeletal traction for a patient. What advantage is conferred by balanced traction? A) Balanced traction can be applied at night and removed during the day. B) Balanced traction allows for greater patient movement and independence than other forms of traction. C) Balanced traction is portable and may accompany the patients movements. D) Balanced traction facilitates bone remodeling in as little as 4 days.

Encouraging bed rest to decrease the metabolic rate Withholding oral feedings to limit the release of secretin Administering parenteral opioid analgesics as ordered

When caring for the patient with acute pancreatitis, the nurse must consider pain relief measures. What nursing interventions could the nurse provide? (Select all that apply.)

Inspection

When examining the abdomen of a client with reports of nausea and vomiting, what would the nurse do first?

"Place pillows between your legs when you lay on your side." "Avoid bending forward when sitting in a chair." "Use a raised toilet seat and high-seated chair."

Which instruction should the nurse include when teaching the client following hip replacement surgery? (Select all that apply.)

D) Clonus

While assessing a patient, the patient tells the nurse that she is experiencing rhythmic muscle contractions when the nurse performs passive extension of her wrist. What is this pattern of muscle contraction referred to as? A) Fasciculations B) Contractures C) Effusion D) Clonus

Approximately 80 to 120 mL

While caring for a patient who has had radical neck surgery, the nurse notices an abnormal amount of serosanguineous secretions in the wound suction unit during the first postoperative day. What does the nurse know is an expected amount of drainage in the wound unit?


Related study sets

AP Biology In Class Final Review (Part 1)

View Set

Economics Chapter 12 Practice Questions

View Set

Newton's First Law Test (Physics)

View Set

Загальна патологія (екзамен-крок)

View Set